Catch up on stories from the past week (and beyond) at the Slashdot story archive

 



Forgot your password?
typodupeerror
×
Education

Uniquely Bright: Experiences and Tips? 1309

An anonymous reader writes: "I would like to hear from fellow /.ers that consider themselves unusually but non-traditionally 'bright' and how you have dealt with it. What are you doing now? What did you do for education? How is your life now? I'm on the verge of entering college, never having liked school much yet always in love with learning. I would like some tips, suggestions, and experience in living with an extra degree of intensity, depth, and general intelligence. I love learning, yet I never have found school enjoyable. I'm incredibly intense and concentrated, yet I often become bored of specific projects in a few months. It's not anything diagnosable (I've looked into it) but more an inherent trait. Academically, I have managed to be alright, but nothing spectacular. Lots of people I meet think I should have a 4.0 easy, but I'm pretty far from it. My interests are broad, from computers (linux/os x/php/mysql/etc) to photography to cookery, I'm creative and technical. Friends and others recognize my strength in these areas. I can't stand being completely technical alone, but I love it in moderation. My attention span is practically unlimited when I am interested in a topic, and I get intensely interested in it. I want to hear from people who share some or all of these traits. I'm just coming up on entering college, so most of my life is ahead of me. I'd like to hear about everything from your education to your career to things you wish you had done differently!" Sounds like an INTP to me.
This discussion has been archived. No new comments can be posted.

Uniquely Bright: Experiences and Tips?

Comments Filter:
  • Best Advice (Score:4, Insightful)

    by Anonymous Coward on Saturday June 12, 2004 @09:42PM (#9409978)
    Be prepared for your spirit to be crushed
  • College (Score:5, Insightful)

    by Anonymous Coward on Saturday June 12, 2004 @09:42PM (#9409981)
    I couldn't take college and dropped out because of my arrogance, similar as yours. As a result I make 12 an hour for computer repair. It's not the boom anymore, kid.
  • Just know this: (Score:5, Insightful)

    by Uncle Gropey ( 542219 ) on Saturday June 12, 2004 @09:43PM (#9409986) Journal
    You are not a beautiful or unique snowflake. You may have to tough it up and take a path that is not enjoyable to you, as most of the rest of us normals have done, and save the soul-nourishing for the weekends and holidays.
  • My suggestion (Score:4, Insightful)

    by Anonymous Coward on Saturday June 12, 2004 @09:43PM (#9409988)
    Get over yourself. Only when you lose some of that cockiness will you begin to enjoy a meaningful and enriched life.
  • by grahamsz ( 150076 ) on Saturday June 12, 2004 @09:45PM (#9410005) Homepage Journal
    I found i really had to try and commit myself at university, otherwise i'd find myself with a final exam the next day writing some random perl code to catalog my music collection.

    If you can channel your energy and focus on the not-so-interesting parts then you should do pretty well.

    Once you're in the real world it's a bit different, but hopefully you can find a work environment that suits you.
  • discipline (Score:5, Insightful)

    by timeOday ( 582209 ) on Saturday June 12, 2004 @09:48PM (#9410021)
    You will have to focus and refine your talents to get anywhere. The ability to work really hard for a short time when you happen to feel like it won't help you any. Otherwise you will feel cheated when those without your "raw ability" whiz by you in life.
  • by odenshaw ( 471011 ) on Saturday June 12, 2004 @09:48PM (#9410024)
    College is so you can show the employer that you can deal with a whole bunch of bullshit... stuff you didn't want to do and still did anyway.
  • by YrWrstNtmr ( 564987 ) on Saturday June 12, 2004 @09:49PM (#9410030)
    ...and then do it.

    A lot of people (especially in here) are going to tell you "yeah, I'm the same...this is what you should do"
    or
    "Shut your whiney cakehole. Go to school, get a job, and go to work."

    All bullshit.

    Sample many things over the next few years, find something you like to do, and then go do it. After that, all bets are off.
    If you can't find something you like to do, something that fits in you mindset at the moment...do something anyway! If it sucks...too bad. You still need to, at the very least, support yourself. Because I won't. And neither will the next guy. And your parents shouldn't have to.
  • by Gyan ( 6853 ) on Saturday June 12, 2004 @09:49PM (#9410036)

    Don't try too hard to beat or overtake the system. It frequently happens that the class/project/whatever is too slow and/or easy for you. Don't get distracted and procrastinate on something else. Societies and formal institutional systems don't give free reign or tolerate deviants too much unless someone in power recognizes your potential and empathizes. There will be a few aspects where you can do as you wish, but not on the whole. It's not very optimistic advice, but it's practical.
  • My experience... (Score:5, Insightful)

    by bishr ( 262019 ) on Saturday June 12, 2004 @09:50PM (#9410043)
    My experience with this is limited, but that a lot of postgraduate education is not set up for your type; they're looking for people with more discipline, who will see projects through to the end and get published (and possibly make advancements in whatever field).

    Your type of intelligence frustrates many people because it's not helpful; to produce usable software or make advancements in practically any field, you need to focus on them for a long time; I'd say that most of the "obvious" or "easy" discoveries have been made, and much of the research out there is fine-tuning what we know.

    The best thing to do is to find a mentor, someone who has a similar mindset. You may find one at your institution, but you shouldn't rule out looking further. In order to do discover or create something important, you need to overcome this... Of course, lots of very effective managers and adminstrators are like this; expand your search for a mentor to maybe the field of business... And check your ego at the door. You may think you're incredibly bright, but just wait until you hit postgraduate education. I'm in medical school, and some of the people around me are exceedingly intelligent, and others are average joes like me. The higher you go, the more you realize you're not "uniquely" anything.
  • Re:Advice (Score:5, Insightful)

    by tha_mink ( 518151 ) on Saturday June 12, 2004 @09:51PM (#9410048)
    The real thing you need to do is get over yourself. You're not special. There's lots of people in this world that are just as smart as you. Once you get over yourself, the world is your oyster. "unusually but non-traditionally 'bright' "...jesus...Kill me. Get over yourself.
  • by Anonymous Coward on Saturday June 12, 2004 @09:51PM (#9410050)
    After you've been out having to work for a living 40+ hours a week, you'll find yourself thinking how easy it would have been to have gone to class, studied, did your homework, and got that 4.0

    It ain't smarts, it's effort.

  • Re:PPPOP (Score:2, Insightful)

    by Gramie2 ( 411713 ) on Saturday June 12, 2004 @09:53PM (#9410072)
    Let's hope that this reader learns some lessons that you apparently haven't.

    Happiness doesn't come from accomplishment, possessions, or the admiration/envy of others.

    Happiness comes from within, and can mean accepting your own limitations, and the limitations of others. If you can't appreciate the qualities of those you term "stupid people," it will be hard for you to accept your own failings.

    Being isolated and miserable isn't a result of "failure to submit yourself to the stupidity of our self-crafted society." It comes from not understanding how to form deep, committed relationships, in not finding joy in small, everyday things that are nonetheless wonderful.

    There is a place for dissatisfaction with the status quo, for striving and achieving things that are new and unique. That's the path you take, not the goal, nor the meaning.

    And always, always accept that sometimes other people do know better, and sometimes you are wrong. Humility is essential.
  • employer that you can deal with a whole bunch of bullshit

    Because you're going to be dealing with bullshit for the rest of your life.

    Sad but true. Find a job that you can live with until retirement. Have interesting hobbies. Don't talk about them at work - work is supposed to be your hobby, and I don't mean that you admit to doing recreational programming. The Boss thinks he owns you 100%.
  • by Dasein ( 6110 ) * <tedc@codebig. c o m> on Saturday June 12, 2004 @09:55PM (#9410087) Homepage Journal
    So, I was in the "gifted" program going through schools and it was all pretty easy for me as well. My big regret is that I didn't work harder at academics to begin with. I ended up getting a job instead. Although I've learned a lot and accomplished a lot, I've always wondered exactly what I missed by staying in school and working really hard. I look back on all the money and career success and I frankly hold it pretty cheap.

    So my advice, is find the hardest major in which you're interested and go work your ass off. Then, when you get to be my age and look back on it, you won't have to wonder because you went all out and did something really hard.
  • by gothzilla ( 676407 ) on Saturday June 12, 2004 @09:57PM (#9410098)
    I was just like that when I was younger. Kept getting put in honors and advanced classes because of my intelligence yet finished high school with a 2.05 GPA because I was so bored and couldn't get into the subjects. Tried college and I just wasn't into it. Ended up joining the military so I wouldn't have to work for minimum wage anymore. Seriously, suck it up and finish college. The regret of not finishing lasts a lifetime.
    I found that spending a lot of time studying things I liked helped deal with the more mundane aspects of life. You don't have to end up in a dull job though, no matter what you like or are good at, there can be good money to be made doing it if you plan and think ahead. Every career has good paying jobs and bad ones, and the good paying jobs are far fewer. Welders can make some money welding mufflers, or they can learn underwater welding and make serious cash.
    Most importantly, don't let anyone tell you what you should do for a career. Most people won't be able to comprehend your situation and offer good advice. I let a girl talk me into being a physics major and even though I was plenty smart enough to do it, it wasn't something I was very interested in and failed miserably.
    Find your interests and follow your heart.
  • I'm with you... (Score:3, Insightful)

    by stevens ( 84346 ) on Saturday June 12, 2004 @09:57PM (#9410104) Homepage

    I was in the same boat. If a topic interests me, I eat it up with vigour. If I'm uninterested or bored with it, I can't even force myself to do it. Result in school? A mix of A+ and C-.

    I went into programming because it interested me. I was lucky that it is also a very unregulated industry--you don't need a string of letters after your name; my Bachelor's does fine. This is important for people like us, because you want a career where knowledge counts but certificates don't (as much).

    My advice is: never stop learning, but don't waste your time with too much school. I declined grad school because I thought I'd die from boredom; but after a few of years working I have a position where I basically get to direct my own work to what I find interesting. Businesses need self-learning, independent thinkers. Trust me, I'm trying to hire, and while there are many "trained" people, there are few with an agile mind and good judgment. We have enough drones.

    Don't drop out of your undergrad--it's great fun! But try to slog through the boredom, and learn as much as possible on the way. Good luck with school, but remember to get out before you lose your mind.

  • by freejung ( 624389 ) * <webmaster@freenaturepictures.com> on Saturday June 12, 2004 @09:59PM (#9410120) Homepage Journal
    You are not a beautiful or unique snowflake.

    On the contrary, you most definitely are. And not only that, but you have a unique and beautiful contribution to make to life, you just don't know what it is yet. Don't worry about that. It will come to you in time.

    The only good advice I can give you is to follow your heart. That may sound trite, but it's true. The Universe is way too complex for even the best brain to control and predict. You never know what's going to happen to you. It's far more important that you do the right thing than it is that you do what is a "good career move" or whatever.

    Don't save your soul-nourishing for anything, get it in everything you do. If what you're doing doesn't nourish your soul, do something else. Don't feel you have to do any particular thing just because it looks like an easy path to money. The most extraordainary things can happen in life, so keep your eyes and heart open.

    "Don't worry if you don't know what to do with your life. The most interesting people I know didn't know at 20 what they wanted to do with their lives either. Some of them most interesting 40 year olds still don't." -- Utah Phillips

  • by Frisky070802 ( 591229 ) * on Saturday June 12, 2004 @10:00PM (#9410123) Journal
    From the story, I'd say "uniquely conceited" is more appropriate. But then, posting this on slashdot ought to take him down a peg or two if he reads the responses....
  • Re:Just know this: (Score:3, Insightful)

    by BWJones ( 18351 ) * on Saturday June 12, 2004 @10:01PM (#9410132) Homepage Journal
    I would have to agree with this. It is also true of kids that are extraordinarily bright and very high achievers early on in life. The harsh reality that we all had to live up to was that you grow up and get a university degree, perhaps a doctorate (or two), and then you are just like everybody else you associate with, only you peaked earlier. I am still in my early thirties, but everybody around me is pretty much equally smart and accomplished. Some are happier than others so the secret is finding you niche, what it is that you like doing for a living and striving to continue making a difference. The really sad cases to me are the ones where folks make a name for themselves early on and then ride on their early efforts for as long as they can eventually becoming bitter.

    On the other hand, you do tend to get opportunities that you would not normally receive, and I would encourage you to explore as many of those as possible. If you don't know what you want to do, go to a liberal arts college and see where that takes you.

  • Re:PPPOP (Score:3, Insightful)

    by Anonymous Coward on Saturday June 12, 2004 @10:01PM (#9410133)
    Or you could be smart enough to realize that the gateway to interesting work is to do the mind-numbing task.

    One of the best things you learn in school is the discipline to carry on through things that need to be done. As a scientist, I can tell you that the ratio of time spent in deep thinking (which is fun) to time spent churning out the experiments and papers (which is often mind-numbing) is at least 1 to 20. If you think of yourself as above menial work, your thoughts will never escape your own head, however brilliant they are.

    You don't have a choice but to submit to the system "crafted by charlatans and intellectual inferiors". The best you can hope for is to jump through the hoops, then once on the other side, order the hoops removed. If you fail to see this, you are not as bright as you claim; while if you do see this and are unwilling to do it, you are merely a whiner, lamenting that the world is not the way you want it. I don't have a lot of patience for that point of view.

  • by Richard_L_James ( 714854 ) on Saturday June 12, 2004 @10:06PM (#9410176)
    I fully agree with you that he should learn to focus on one thing. Or to put it another way it is easier to normally go with the flow than to try to swim up river (well initially anyway). However I do NOT agree with this statement:

    I'd say that most of the "obvious" or "easy" discoveries have been made, and much of the research out there is fine-tuning what we know.

    There are still many obvious discovers stil to be made for the simple reason that so many people these days are busy working complex ideas that people have forgotten that very simple ideas often make a big difference, e.g. wheel, axe, chalk, paperclip.... Hmmm, paperclip maybe I should have left that one out ;-)

  • Thank You! (Score:3, Insightful)

    by bobobobo ( 539853 ) on Saturday June 12, 2004 @10:08PM (#9410184)
    That was exactly what I was thinking when I read the article blurb. Typical, "I read slashdot and use a computer therefore I'm a unique genius."
  • by potus98 ( 741836 ) on Saturday June 12, 2004 @10:08PM (#9410185) Journal

    A future PHB? All the ingrediants are there: arrogance, cockiness, self-delusion... Kid, you're headed for middle-management!

    These are the types of unrealistic self-loving kids you get when all you do is shower little Johnny with positive reinforcement no matter how much he sucks at [fill in sport, hobby, or interest here]. Chances are, this kid attended government schools. And now he's comparing himself to those teenagers? Maybe you really are special, though statistically speaking, I doubt it.

    You think you're bright, sharp, and multi-talented? Anyone can have that impression when they compare themselves to their coincidental surroundings (family, local friends, etc.) Think you're good at computers? Go to Berkeley or someplace where you will really be challenged. Like rockets? Get a PhD and join NASA. Great swimmer? Then get on the Olympic team. Otherwise, you're just another schmuck.

    Don't get sucked into comparing yourself against easy targets like teenage pals. Until you work with the best in a given field (or even the pretty good) you have NO idea how much you suck.

    And if you're good at cooking, go win an Iron Chef tournament. Until then, reel in the ego before you get pounded.

  • by Punk Walrus ( 582794 ) on Saturday June 12, 2004 @10:08PM (#9410190) Journal
    Yeah, and I bet you are very similar to many Slashdotters in this. I am exactly like you describe; easily bored in topics I hate, and intensely focused on stuff I love.

    In school, I was always considered bright and gifted, but got a lot of comments that I never applied myself, or "reached potential." I despised that kind of comment, because how the hell would you know what my "potential" was? But I grew up in an abusive home, and to make a long and complex story short, by 18, I was living on my own.

    This was my true test, and I did pretty good. I went from being essentially homeless to living with friends to getting a job, making friends with cool people, and while I can't say it's been an easy or the best life, I'd say for me it turned out pretty well. Looking back on it, college and I would have never gotten along. I have always hated structured and abstract learning (meaning "learn this way, and we won't tell you why, or how anything relates to the real world cases!"), and I got accused of daydreaming by some teachers and "asks too many irrelevant questions" by others. Guess which classes interested me?

    My "self-education" led me to computers, and the drive to learn how things worked made me a better and better tech. Soon I worked at a call center, and kissed my retail days good-bye. Then I was doing QA. Then I was programming call centers. Then I was working an International help desk for a large ISP. Now I'm managing proactive QA Testing solutions that keep the Internet going for millions of people. Never had a college degree, but I have certifications and company awards on my walls. I love what I am doing.

    I didn't gain anything by being an office backstabber, either. I found you gain more opportunities with friends, so I make friends wherever I go. And I have found that is the key to being successful in any career is the connections between people.

    My advice to all young people of any career or life path is to make friends, be friendly and polite as much as you can, learn people's names, and never look down on anyone, no matter how "insignificant" or "a jerk" they seem. Learn from them. That may be "just the janitor" but he has keys to rooms, you know what I mean? Humans are social beings. They love attention. If you give them attention, they seek you out. And never forget those who have helped you in the past, either.

    As the saying goes, "It's not what you know, but who you know."

  • Re:Just know this: (Score:3, Insightful)

    by Anonymous Coward on Saturday June 12, 2004 @10:10PM (#9410202)
    You're exactly right. I read his blurb, and I can immediately think of 20 people just like him.

    Everyone gets tired of projects after a couple of months. Very few people see projects through to the finish without getting burnt out on it.

    Lots of people have hobbies and interests. Most people who are in professional fields, and who are good at it enjoy learning as much as you, and have just as many hobbies.

    Now the important part: Everyone has to pay their dues.

    I think Engineering is the perfect thing for you. If you get in at the right place, and pay your dues of hard work, and pushing through getting bored/sick of projects, then you'll be good. I'm in the middle of paying my dues right now, but I'm also seeing the benefits. The higher levels of engineering are the "brain trust" levels.

    One Prof once told me that your goal as an Engineer should be the guy that forms the projects. Be the idea guy, be the brain trust of the company. At that level, you work on a project brainstorming all the really neat and revolutionary ideas for a couple of months, then toss it off to other engineers who do the nitty-gritty detail stuff. In the mean-time, you move on to another project.

    It's a great place to be, but you don't get there without paying your dues. Dues include going through college, masters, maybe PhD, then doing quality, grueling work for half a decade.

    Anyways, good luck. Eventually you'll realize that there's a whole big chunk of the population just like you, and they make up the people at the creative tops of most industries.
  • Re:Bwuhhaha! (Score:1, Insightful)

    by Anonymous Coward on Saturday June 12, 2004 @10:11PM (#9410216)
    I think I have to agree with sumac. Your question for the Slashdot community et al probably won't get you much help but it is likely to inspire anger in a lot of people (as you came off arrogant) and, in my opinion, ignorant. It seems to me that you might have ADD, as in your question you repeated the same thing a few times, which unfortunately made me pretty sure that in fact you are most likely an average Joe and not this genius you are trying to hype yourself up to be. Then again, you might be gifted, and that's just fine. The fact of the matter is, you won't get very far in life if you keep touting that you're "Uniquely Bright." It's probably been said a million times already, but you really do need to STFU, get a job, and stop wasting people's time with your "Uniquely Bright" bullshit. On a related note, why in the hell did this make the front page of Slashdot? I think a front page GNAA post would be much more interesting than this narcissistic flamebait. Where are the troll groups when you need them?
  • Heh. (Score:5, Insightful)

    by ivern76 ( 665227 ) on Saturday June 12, 2004 @10:15PM (#9410239)
    My only advice would be to stop telling people you're "uniquely bright." It doesn't go over well. That kind of thinking is something you just keep to yourself...instead, demonstrate your intelligence through action. If you're interested in programming, I'd suggest picking a pet project and getting your hack on.
  • Go to work.. (Score:1, Insightful)

    by Anonymous Coward on Saturday June 12, 2004 @10:18PM (#9410262)
    I had the same type of attitude toward school. I went to about 2 years of college, and decided that it wasn't my thing. I was too busy working on other projects, and wasn't spending enough of my time on school. I stopped going to college, and went and got myself a great job with all of my skills as a sysadmin. After a year or so the company I was working for sent me back to school, and paid for my tuition to boot.

    I didn't want to look back at my life and tell people that I'd only been through a couple of years of college. Believe it or not, it does make a difference out there in the real world in a lot (but not all) of cases.

    After taking those couple of years off, I found that I had a lot more tolerance for school. Where I was an average 3.0 student before, I became a straight 4.0 student through the rest of my degree. Sometimes it just takes a bit of maturity to help learn how to hold that attention span, and push you to make that extra effort to make everything you do be the very best you can make it.

    My advice: go to school, work your ass off in every single class no matter how un-interesting you might think it is. Don't be one of those students joining in the "this class is too hard" chorus. Prove them wrong by showing how "gifted" and bright you really are.

    If you're up to it, find a flexible job, and work full time, and go to school full time. That way you'll get the best of both worlds. That's what I did, and it has opened a whole new world of opportunities to me.

    Best of luck!
  • Get your degree (Score:2, Insightful)

    by Salo2112 ( 628590 ) on Saturday June 12, 2004 @10:22PM (#9410290)
    I hated school so much I refused to go to college out of HS. I worked through a string of low-paying "shit" jobs. Funny thing, I was usually much smarter that the people in charge, but I was making less than they were and they were telling me what to do. They had degrees,

    One day, I got lucky and got a job that paid quite well - an industrial manufacturing job. My first day on the job, an old man walked up to me, asked me how old I was and then told me to look good and hard at him. He was old, worn out and broken down - a life of manual labor had taken quite a toll on him. It was his last day there - he was retiring. The old man told me not to let what happened to him happen to me.

    That night, I applied to a local community college.

    It took 10 years to get my bachelor's degree (night school and working a full time job to pay for it). I missed out on a lot of "fun." I wasted a lot of time I can never get back.

    Suck it up, and get the degree.
  • Re:Just know this: (Score:1, Insightful)

    by Anonymous Coward on Saturday June 12, 2004 @10:22PM (#9410293)
    I disagree. Much like the original poster, I too was a child prodigy... I was diagnosed with an IQ near 180 when I was a child (which was a relative diagnosis - I'm sure). I spent my life in "gifted" and "accelerated" classrooms. I got less than average grades, Bs, Cs, and Ds; however I still graduated high-school (with a few AP credits under my belt, I might add). I attended a First Tier College and after a few years there, I dropped out.

    I dropped out because of lack of interest...

    Ultimately, I went on to become a wealthy serial entrepreneur by persuing my ventures 100%. Entirely too many people "rely" on their degrees. They tend to think that just by having one they have "arrived". The reality is that what you "do", is far more important than what you can convince others that you've "done".

    High school sucks, period. But definitely go to college. College is a great experience and it's much better than high-school. You will learn some insane things. Granted, right now you think you know everything; however, when you're sitting in stadium sized black room full of humming supercomputers (hint: SCRI), you will really appreciate your decision to go to college. If you decide to finish or not, is your decision. The experience alone is worth the price of admission. What you do after you're "experienced" is what truly matters. In academia, finishing school is a de-facto measure of accomplishment.

    In the real world, what you DO is the bottom line.

    Find a project you're enthusiastic about and persue it to completion.

    The hardest part is following through.

  • by Anonymous Coward on Saturday June 12, 2004 @10:24PM (#9410311)
    The thing with school is that you really don't have a choice as to what you learn, you just learn.

    and there's a reason for this. The solution to problems in life may require skills that you don't see a use for or have any interest in when you are learning them. I spent a lot of time thrashing in college until I just knuckled down and learned everything they threw at me, whether interesting or not, whether I could see a valid use or not. I had to or I just wasn't going to graduate.

    30+ years later, it is amazing to me how much that I had to force myself to learn was not only useful but vital in solving problems that I wanted to solve. In fact, some of them were things that I had to relearn because learned them poorly or not at all in college.

    With the limited perspective of a college student, there is simply no way that they can predict what may be necessary later in life.
  • Re:Advice (Score:5, Insightful)

    by ldspartan ( 14035 ) on Saturday June 12, 2004 @10:24PM (#9410312) Homepage
    Yes!

    Not so much that exactly, but don't think you're a fucking genius because you're the smartest kid in your highschool. It takes a lot to be succesful, intelligence alone won't do it. I'm sure most of know (or are) plenty of very smart people who are not effective in the real world because they can't communicate effectively.

    College is a totally different story, particularly if you're going to a good one. You'll meet people there who are intelligent, motivated, and personable.

    Also, if "non-traditionally bright" is a cop-out for not working hard enough to fulfilly your potential, well, good fucking luck. If you find your undergraduate degree easy (as I have to a large extent), you probably should have gotten into a better college. If you're in one of the best colleges for your degree, you should probably be getting another degree. If all those things are true, get a graduate degree.

    --
    lds
  • by workboomer ( 738265 ) on Saturday June 12, 2004 @10:25PM (#9410317)
    I think what this person feels is not uncommon. A lot of us think we are smart because we only know ourselves... we don't know the competition!
  • by Transcendent ( 204992 ) on Saturday June 12, 2004 @10:26PM (#9410325)
    Don't be arrogant.

    You'll go out and find you aren't as special as you think you are. Yes, I was the same way... and still am. I have a 3.92, graduated HS with lots of honors (higher gpa though... could have been higher if i didn't slack off my soph year), got all the comments about intelligence, genius, whatever. Now i'm a junior in college and work too.

    I have a co-op job and work with some people I concider not as intelligent as I am or they just don't grasp things like I do... but I don't care. They know things that I don't, they think differently than I do, but they're engineers, they're smart too. You can be gifted in many different ways (I used to work at a bagle shop and had an awesome General Manager there. He was gifted in his own right and I highly respect him).

    At work I have it setup where I get daily dilbert when I log into my computer... odly enough last week was pretty much all about the "prima donna" of the office. And now my only advice to you is to try not to be that guy.

    Even in classes the same rule applies. Don't be that guy who thinks he's god's gift to the classroom/lecture hall. I've seen plenty of those, and no one likes them.

    Who did we like or admire? The guy who got close to the higest grade on all the exams but kept to himself. He was bright, and not an ass hole.

    Sure you might have the gifts for science, computers, art, music, or whatever you like... but what you really need to keep up in the real world is to have the social skills.
  • by metacosm ( 45796 ) on Saturday June 12, 2004 @10:28PM (#9410338)
    Article: "Unskilled and Unaware of It: How Difficulties in Recognizing One's Own Incompetence Lead to Inflated Self-Assessments" [apa.org]

    abstract: "People tend to hold overly favorable views of their abilities in many social and intellectual domains. The authors suggest that this overestimation occurs, in part, because people who are unskilled in these domains suffer a dual burden: Not only do these people reach erroneous conclusions and make unfortunate choices, but their incompetence robs them of the metacognitive ability to realize it. Across 4 studies, the authors found that participants scoring in the bottom quartile on tests of humor, grammar, and logic grossly overestimated their test performance and ability. Although their test scores put them in the 12th percentile, they estimated themselves to be in the 62nd. Several analyses linked this miscalibration to deficits in metacognitive skill, or the capacity to distinguish accuracy from error. Paradoxically, improving the skills of participants, and thus increasing their metacognitive competence, helped them recognize the limitations of their abilities."
  • by Squeamish Ossifrage ( 3451 ) on Saturday June 12, 2004 @10:29PM (#9410344) Homepage Journal
    For pete's sake: You can't take a word that's in common use, re-define it, and expect people to take it up. Language change happens, but you can't force it.

    It also doesn't help when your re-definition is absurdly conceited. You're essentially claiming that yours is the intelligent position by appropriating the word. It would be Orwellian if it weren't ridiculous.

    ...And this is coming from someone who agrees with the principles "Brights" espouse.
  • by Anonymous Coward on Saturday June 12, 2004 @10:30PM (#9410350)
    I don't know if I'm in your category, but here's what I did:

    W R O N G
    =========

    1) Too much focus on getting grades; I should have worried less about exams and more about learning.
    2) Too much attention to classes, didn't have a life until 17.
    3) Too much attention to life (friends, cars, women etc.), totally blew school around 18; I should really have used better my time.
    4) Excessive respect for teachers: usually the harsher ones were NOT my friends... just filled with prejudice and hatred.
    5) Lack of consideration for some great dudes who were my professors: some I let down when I dropped out of their courses, with others I argued too much without trying to understand their views (and learn what they tried to teach, God bless them).
    6) Excessive respect for my family: trying too hard to please them, by doing things I would never do, because I REALLY couldn't force myself.
    7) Lack of maturity to do the right thing in consideration of my family's best interests, even when that meant to choose a different path from what they had planned.

    R I G H T
    =========

    1) I always loved to learn, even when I was lazy to do my homework.
    2) I always wanted to accomplish great things, and I still do... with age, though, I got more realistic about what I can do.
    3) I always had faith in myself, even when most wouldn't... Homer is my hero ;-)
    4) I did start things all over again, and eventually was succesful. Winning after having lost is much better than winning the first time.
    5) I had an excellent course at the university, because I made it excellent. I enjoyed every single minute. I marvelled at how things were fascinating, and I did make questions all the time, even when everyone was almost asleep (at 11 PM).
    6) I learned to use the school as a tool. Do you know what is important to learn in an area of knowledge? What to learn first? Which are the dependencies? How to organize classes? Well, the school has all this ready. Not always well done, but usually good enough so that you can focus energies in learning.
    7) I learned to "work together" my classmates... being a leader is ok, but sometimes you're chosen because you have the best handwriting. There's really no "lesser guy", I concluded. This led to a more productive life for me in several environments, including with my wife. With father, mother and brothers not, but that would be asking too much. ;-)

    Hope this helps.
  • by ploppy ( 468469 ) on Saturday June 12, 2004 @10:30PM (#9410353)
    I was like you as a teenager, interested in everything, the occult, radio theory (early radio design of the 20s/30s), computing, history. Deep interest for a while then complete boredom until the next interest. Always an obsession with one then moving onto the next, to the exclusion of other interests.

    School work (including the degree) was always a struggle, never any interest, it was either something I was not interested in (having strong views on everything), which meant it was a struggle to do well, or it was something I was interested in, which meant I'd already done it to a far greater degree than it was taught, leaving the teaching boring slow and patronising (as I thought then). Either way I always found it difficult to do as well as people who I considered of lower ability (in my not very humble opinion then).

    The PhD was completely different, no taught component (British PhD), just an ability to spend three years obsessionally doing something I wanted to do. In the process my academic performance changed from so-so (upper second degree rather than the 1st I was easily capable of), to PhD work which was (in my opinion and others) one of the best and most innovative PhD research produced in the department.

    The difference? Not having to deal with slow teaching, accomodate other people's views and being able to do a very individual PhD.

    The problem (as I realised later) is all those pieces of paper are important to get because no body is going to take any notice if you've not got them! I always hoped my brilliance and clever projects undertaken in preference to the school work would be recognised, rather than the boring pointless assignments and exams. They were not. I learnt I should have treated all the taught work as a game, play the game and get the marks.

    The problem post PhD is that everything is as plodding and constrained as the degree. Become an academic, be prepared to put up with lots of rules and regulations, pointless teaching (I found setting assignments and exams as pointless as I found taking them as a student), corporate IT is of course even worse.

    Your intelligence and iconoclasm will always make you an outsider and misfit. Ultmately you have to learn to accept it, and know when you have to play the game even when you don't want to.

  • Some real advice (Score:5, Insightful)

    by Scratch-O-Matic ( 245992 ) on Saturday June 12, 2004 @10:31PM (#9410354)
    I felt the same as you...maybe I'm as smart as you and maybe not.

    But here's the thing that has made my life invaluable. No shit...

    FIND AND CHERISH GOOD FRIENDS.

    Your talents will take you wherever they take you, but friendship will fill the gaps.
  • Re:Best Advice (Score:5, Insightful)

    by nycsubway ( 79012 ) on Saturday June 12, 2004 @10:36PM (#9410391) Homepage
    Be prepared for your spirit to be crushed

    After that happens you'll understand what you're really capable of, and you'd be surprised how little you know now. You'll start to understand what all the 'old people' on slashdot are waving their canes about too.

    If you're realisticly confident in your own abilities you have a chance of having them happen. Life is a lot about working and trying hard. You will have to work for everything you ever do, and if you dont work for it, you wont be able to appreciate it. If you're confident and you understand that you need to fail to get better, you have a chance of becoming what you think you can be.

    As a side note, its a pretty naive and narcissistic view to feel your all that and a bag of chips.

  • by Anonymous Coward on Saturday June 12, 2004 @10:36PM (#9410392)
    Sure, just like you I love to learn (everything from Linux to religious history, from why some fish change colors to the artworks of DaVinci, from relativity to inmortality, and everything in between), and learning anything for me is easy (like English), and I can solve most problems of all types in my head almost instantaneously by using simple patterns, but that a genius does not make me, or make other people less smart than I am in many areas.

    Long ago I had a friend who to the eyes of most people was mentally retarded (I mean, this guy took the SAT three times just to get a way-below average score...), but if you gave this guy any video game in existance, this guy would beat it in a couple of days. And I don't mean just action games, but logic games with puzzles, simulations and so on. He couldn't figure out how a REPEAT loop worked in BASIC (I'm not kidding), and yet he could kick my ass on any game any time. Like him, I've met many people in life who at first seemed "not on the bright side", but once you get to know them you'd be amazed. I think intelligence is a relative term to what you're doing and (most importantly) to what you like (or in some cases, what your particular brain "wiring" is best at). The fact that people like you and me like so much to learn and are good at many things does not necessarily makes us brither or smarter; it's just a different life; everyone thinks different and has his/her own motivations and tastes in life and how to approach those things.

    So I'm happy that you love to learn, you hate school but are probably great at the things you like, but before you (consiously or unconsiously) think you're brighter than the rest, look around again in more depth, and you will realize that the fact that you lack the virtue of finding out what other people's great qualities are, is something that from someone else's point of view can place you in the "less bright" category (although, as I explainined above, that'd be wrong too).
  • by Bishop ( 4500 ) on Saturday June 12, 2004 @10:43PM (#9410438)
    At its core the book How to Win Friends and Influence People is a book on social engineering. It is study on hacking society. Every geek should read it.
  • by twitter ( 104583 ) on Saturday June 12, 2004 @10:49PM (#9410474) Homepage Journal
    I read and like "How to stop worrying and start living." It has much of the same sound advice.

    I don't like agreeing with the above post because it's obvious flamebait. "teenage nitwit," is the most obvious attack. The straw men constructed show the author advocates kindness more than he practices it. Garyok, how the hell do you know how our anonymous reader treats people or tells them? How do you know that they are not really clever? As you say, "people deserve respect".

    That being said, my best advice is to get over being clever. I gave myself lots of problems before I did this for myself. I was self defeatingly lazy about the way I did my work.

    Every little thing counts. More than anything else, your school work shows that you can follow directions and are willing to do things that are boring to get what you want. Companies want employees that do everything they are told, not just the "exciting" things. Yeah, it's stupid but that's the way the world is made. You may not like working for a company that judges people this way, but most are like that and it beats being unemployed.

    The most important thing for my technical work was to see good examples. The Given, Find, Solution method is the best way to avoid mistakes and it really saves time even for trivial problems. Trivial problems don't require as much write up. You don't have to be a neat fanatic about it or even have good penmanship, but stating all of your assumptions and referencing equations and other sources makes your mistakes obvious to you when go back to check it. It gives you time to clear your head and avoids transcription problems because you can put your finger on your work and in the book at the same time to check. It also gives you a body of work to take to interviews.

    Look for other bright people and work with them. It will help you understand just where you fit into the world and you will understand more. I picked people at random and did well with one or two of them. One of them is still a very good friend and I have no idea why he thinks I'm brighter than he is.

  • Own business (Score:4, Insightful)

    by cubicledrone ( 681598 ) on Saturday June 12, 2004 @10:52PM (#9410493)
    Unusually bright people are not welcome in day to day society.

    Popular culture goes out of its way to make people who are intelligent, well-spoken and aware appear to be strange. Very often those people are excluded as quickly as possible from the societal "cocktail party on the patio deck."

    The reason for this is that people who are not unusually bright mistakenly believe that intelligent people make them look stupid. The intelligent people must therefore be removed from the stage as quickly as possible, otherwise they might begin to have some political influence which would reduce middle management's ability to stuff their own pockets.

    Smart people are the first ones ridiculed, the first ones argued with, the first ones made fun of, the first ones fired. Smart people often have little or no use for office politics, which is why it is so easy for lying cheat fuck middle management bastards to outmanuever them and get them fired.

    Bright people usually begin their careers believing the quality of their work will enable them to succeed. What they later find out is that there are two choices: spend your career wading through a swamp of bubbling, wet shit with liar cheat rat bastard fuck "supervisors," or start your own business with a couple of other bright people and bypass the cubicle bullshit factory. The quality of someone's work is absolutely irrelevant to success in the workplace. In fact, the higher the quality of someone's work, the more likely it is they will be fired.

    Business encourages office politics and people who are liar cheat fuck bastards always win. Bright people mistakenly believe that being a liar cheat fuck bastard should disqualify someone from competent professional discussion. It does not. In fact, it usually gives the liar cheat fuck bastard an insurmountable advantage. So, the smart people get fired, leaving entire floors full of liar cheat fuck bastards who are paid exorbitant amounts, do no work, yet can't be fired because they have mastered the arts of office politics and being a liar cheat fuck bastard.

    Mediocre, visionless, imaginationless, dull people are usually the first to buy a home, first to raise a family, first to get promoted, first to drive the expensive car, first to put in a pool, first to take the vacations. They can't be fired either, because they never say anything except "there's cake in the conference room" and "are you on the morning donut list yet?"

    So, if a bright person expects to enter the workplace, expect to find four groups:

    1) Upper management, pockets already stuffed with tall dollars, ordering lunch from a golf cart, oblivious

    2) Middle management, busily stuffing their pockets with whatever is left over, ordering in from the local delivery deli.

    3) Dull, witless drones, talking about their weekend trip to "the river" or "the canyon," what color their new Navigator will be, and the landscaping on their palacial four-acre estate, financed because they have never been fired, ever, and ordering lunch as a group from the menu at the local "yuppie grill" which is the only place in town where one can order an $11 bacon cheeseburger. They can afford it, after all, because they have never, EVER opened a bill without a matching paycheck.

    4) One or two smart, intelligent people, quietly working through lunch on a brilliant project, unaware they will be fired a few days before or after it is completed.

    I have long since given up on the "job market," because after three and a half years of being unemployed, and over 400 resumes, I believe it to be a festering maggot-infested open sore on society, draining every last shred of joy and wisdom from people's careers, and destroying the educations and communities of millions upon millions of hard-working people.
  • Re:Advice (Score:4, Insightful)

    by Zeinfeld ( 263942 ) on Saturday June 12, 2004 @10:54PM (#9410498) Homepage
    The real thing you need to do is get over yourself. You're not special. There's lots of people in this world that are just as smart as you. Once you get over yourself, the world is your oyster. "unusually but non-traditionally 'bright' "...jesus...Kill me. Get over yourself.

    Smart is not always what succeeds. There are plenty of stupid people that get far. Take Dufus in the Whitehouse for example. Few people would claim that he is an original thinker or highly knowledgable. On the other hand there are people with blistering high IQs and degrees in Nuclear Physics who can't find a job better than part time computer class instructor.

    Being intelligent does not count for very much, not unless you are actually prepared to do some work and learn something that is useful. Its like having the worlds fastest computer and no software.

    The worst thing that can happen is if you get the idea that you are so smart you don't need to bother knowing anything. There are plenty of people like that in the world. And even if by luck or family connections you happen to get a great job, you are even worse off because that attitude usually leads to failure.

    That said, there is plenty of stuff that schools teach that is entirely useless. I never saw the point of learning French, the effort required was vastly disproportionate to the benefit, and I have lived in the country for two years. But it is highly unlikely that you are 'differently smart' if you are getting straight Ds and Es in all your courses.

    If you really are gifted you don't have to ask slashdot for career advice, you know what you are best at. I have always excelled at tasks that require analytical reasoning and interpretation of data. Subjects that require rote memorization have rarely interested me. I can write a pretty good history essay with access to reference materials, but remembering the date of the Tolpuddle Martyrs is utterly pointless as far as I am concerned.

    If all you are good at is memorization then you are definitely having to do a lot of work to get anywhere.

    You still have to do work if you are good at analysis, but it is more likely that it won't appear to be work. I tend to think I am pretty lazy, I use the fact that ideas seem to come to me effortlessly to avoid having to work as hard as I should. But most people arround me think I am a workaholic who never stops because I am always doing stuff - the stuff that does not appear to me to be work.

    The brain is like any other muscle, you have to exercise it to keep it fit. I may not spend all my time thinking on work, but I spend almost all my time thinking about some problem.

  • Re:Advice (Score:5, Insightful)

    by Epistax ( 544591 ) <<moc.liamg> <ta> <xatsipe>> on Saturday June 12, 2004 @10:56PM (#9410513) Journal
    That's a little rude. Accurate, but a little rude.
    Here's a more correct message:

    There are people who by their upbringing (religion, social forces, parents) are predisposed in any of many ways that limits their own abilities significantly. You are not one of them. Anything you do, you will enjoy and will be good at. Note that I did not say great, I said good. Great takes work. So you have a few choices:
    Do anything you want, enjoy it, and live off it.
    Work very hard suffering mentally at first, and eventually have big payoffs (psychologically and likely financially).
    Work very hard suffering mentally the whole way. Rewards come but you push them away because they just interfere with what's important.
    Resign yourself to become just a mindless peon.

    Most people are dead long before they've died. They might marry, have children, even occasionally read a book but they are acting on external stimuli alone and are, to be blunt, nothing more than walking sacks of chemicals doing everything they should do. They've lost the spark entirely.

    I am just as guilty as the next person in causing my own undoing. I do take a little from the fact that I at least acknowledge it, and try to fight it. Who knows, even I may not end up useless after all.
  • Let's see (Score:3, Insightful)

    by AdamHaun ( 43173 ) on Saturday June 12, 2004 @10:57PM (#9410515) Journal
    All right. So you're an intelligent slacker who wants to get away with not doing anything you don't want to do. Okay, I can relate. Unfortunately for both of us, unless you were born with a silver spoon in your mouth you probably won't have much success. Allow me to suggest a few compromises that might make your life easier.

    If you want a nice job(and if you're going to college, I hope that's what you want), then you might want to:

    * Drop the elitism
    "Does not fulfill potential", as one poster put it, is a synonym for "useless". If you want to be anything other than a hermit, you need to learn how to adapt yourself to the world. This doesn't mean surrendering your individuality and becoming a tool of The Man, but it does mean that you will have to do things you don't want to do. Trying to pass yourself off as too intelligent and "non-traditional" for everyday life is going to do nothing but piss people off. How would you feel if someone told you that they were too smart to deal with you?

    * Figure out what you want to do
    Being interested in many things is good, but if you want a fun job you're probably going to have to specialize in something too. Make sure that what you major in is what you actually want to do. Internships and co-ops are one approach to doing this. You should also consider what kind of standard of living you want. If you can't handle $30k/yr and no possibility of advancement, then perhaps that degree in Jamaican Basket Weaving is not for you.

    As far as learning goes:

    * Get used to doing things you don't want to do
    Most(all?) decent school require you to take a core set of courses before you get a degree. Each major will of course have its own set of requirements. Some of these will not be fun. Deal with it. You cannot study anything in depth without having to deal with a few unpleasantries. More importantly, it'll make you a better person. Every new thing you learn makes you better at learning in general. Someone else said it better than I did:

    "It's weird how when I look back at college, I find my best compsci

    teachers were, indeed, the most literate teachers. There was one guy
    who read all of Dickens every year. Another guy taught himself a new
    language every year. I remember I happened to be in one of his
    courses during the year he was learning Latin and had to put up with
    loads of these weird Latin quotations he'd put everywhere. Flash
    forward ten years and I'm stuck in a super-intense Latin 101 course
    for grad students who need to learn a foreign language pronto,
    and I realized why my little bald compsci teacher was so gungho for
    conjugation and for quoting Virgil at every turn: you realize that
    in some weird -- perhaps even unconscious -- way everything that you
    force yourself to learn *outside* of your chosen "track" actually
    feeds *into* that track and makes you wild, creative, and utterly
    un-fucking-predictable. You scare yourself, scare your friends, and
    you realize, damn, dude, just chill. Cool it on the caffeine and
    espresso because if you get too juiced with the creative jazz -- if
    you make too many connections -- leaping from liberal arts shit to
    comp-sci shit to physics shit -- it's almost overwhelming. The more
    you learn, the more connections you can make -- and the more
    creative you become.

    On a more practical level, learning to do things you don't like in college will make it easier to do so at other, more important times.

    * Grow as a person
    While it's fasionable on Slashdot to lament one's school years as a waste of time, the truth is that once you graduate you won't have as much free time as you used to. A full time job will take a very large chunk of your energy, energy that you had previously put into hobbies and leisure. Spend your college years making friends, trying new activities, and learning how to live as an adult. If there's anything you've always wanted to do, like play a musical instrument or le

  • Make your own path (Score:3, Insightful)

    by humankind ( 704050 ) on Saturday June 12, 2004 @10:57PM (#9410523) Journal
    This is a great topic. Not because of the subject matter, but the in-between psychology that comes into play here.

    Let's face it, most of the greatest minds of all time withdrew into themselves; strived to be more than what was expected of them because of the adversity they faced and/or their own insecurities. It goes without saying that a lot of people will find your query narcissistic and arrogant and fire off appropriate retorts.

    I haven't read the responses. This is such a cool thread that I feel compelled to offer my own commentary untainted by what others may have said.

    First off, who gives a fuck what anybody else thinks. If you believe in yourself, fuck everyone else. 99.9% of the time people seek to label those as arrogant, anyone who might appear to be more self-confident than themselves. That's not your problem.

    This whole argument isn't about intelligence and wit. It's about self-confidence. The reality is that you're not exceptionally gifted in the physical sense. You can't do anything any other humanoid can do. But you may be more aware than most that the limitations imposed by society are not insurmountable. That's what's special -- not you.

    If you believe in yourself and have been able to demonstrate to those around you that you can excel beyond the mundane, then you don't need to prove anything to anybody other than yourself. What you do with your career is peripheral to what you want to do for yourself. All the great people of the world followed their own path, and they felt confident that whatever they were doing, be it investment banking or brick-laying, they were the best of their kind. That's the way to do things.

    Figure out what makes you happy. If you really want to believe in yourself, fuck college. If it doesn't jive with your dreams, don't do it. College will only serve to make you conform to the roles that others on the assembly line think will guarantee them a career. If you truly are "special" then no matter what you do, you will succeed. The easy way out is to follow the path of everyone else.
  • by chreekat ( 467943 ) on Saturday June 12, 2004 @11:03PM (#9410557) Journal
    I felt the same way you did at the end of high school. I'll tell you why: high school sucks. It is the worst possible way for everyone, and especially brainiacs, to get prepared for life. At the end of high school, I figured I could do anything, and that my intelligence gave me special privileges. Sure, my GPA wasn't awesome, but that's because I knew better to dirty my hands in this system for inferiors.

    College came around and crushed, humbled, and rebuilt me. Now, rather than SAYING I can do anything I want to, I actually CAN do many of the things I want to, and I also know what those things ARE. I found out that lots of people who didn't seem 'smart' in high school are actually really brilliant, and that my smarts don't mean anything unless I apply them. I learned the world will not give me a cookie every time I tell myself, "but I'm smarter than these people!" I learned that I can't wing any and all tests that come my way (I just got smashed in a Numerical Analysis final.. sheesh). Haha, that reminds me.. if you go to college, do your homework! Man!

    The Army bases its ad campaign on that sort of transformation. I'm sure there is truth to it. The bottom line is that you have to _get out and see what life is really like_. Throw everything from high school away. Realize that you are NOT one of the elite 2%, or if you are, that .02 * (population of world) is a big fucking number.

    -b
  • Re:Advice (Score:5, Insightful)

    by cgenman ( 325138 ) on Saturday June 12, 2004 @11:07PM (#9410582) Homepage
    There's lots of people in this world that are just as smart as you.

    The thing that separates intelligence and genius is a lot of disciplined, tiring, rigorous work. And by tiring, I mean staying at the lab for days straight while your roommate calls the cops thinking you have been nabbed. The trap I see many "unusually bright" people fall into is that because everything came easily to them in High School, they never learned to really work at things. But really working at things is how you get somewhere in life... really working at things is how you separate the unusually bright and kind of good from the unusually bright who dedicates their life in a sheer bloody minded pursuit. Simply being brighter and better than most of the people in your High School isn't even enough to avoid crappy jobs... a friend of mine memorized my set of Encyclopedia Britannicas in the span of two weeks, yet jumps from crappy retail job to crappy retail job because he just doesn't "enjoy" doing anything. In reality, he's not dedicated enough to get beyond crappy jobs and into something that he would like doing.

    We all say that intelligence is the highest achievement, but that's not entirely true. Intelligence is distinct from knowledge, which is distinct from dedication. All three are necessary for success.

    If you want my advice, do a trial by fire. Do something REALLY hard and unpleasant, like outward bound [outwardbound.com], the AIDS ride across Alaska [inter.net], or spend a summer of thankless backbreaking toil on an Alaskan fishing boat. Ultimately, you will be glad you did.

  • by Mustang Matt ( 133426 ) on Saturday June 12, 2004 @11:08PM (#9410592)
    I wish I had never attempted college. I threw my money down the drain repeatedly.
  • by chia_monkey ( 593501 ) on Saturday June 12, 2004 @11:08PM (#9410595) Journal
    I too thought (well I guess I still do) I was incredibly bright and talented. Straight As in high school without even trying really. Head on out to Carnegie Mellon and WTF...I'm not the brightest anymore. In fact, I had to bust my ass to be just above average. Since then I've worked a myriad of jobs and started a few companies.

    The point is, go get humbled. Find out where your strengths and weaknesses are once you're thrown in with the cream of the crop. You may find you're in the top 5% when it comes to coding but the bottom 5% in communication and reasoning. If, after four years you still find you're a genius, go out in the world, say you got superior grades at a top notch school and do whatever the hell you want. My guess is though, you'll be eating a little humble pie for the first couple semesters at school.
  • by damiam ( 409504 ) on Saturday June 12, 2004 @11:13PM (#9410624)
    What exactly do you think "profoundly gifted" means? That someone can do well on tests? That's the most meaningless criteria ever. Washington, Marx, Edison, Churchill, etc. weren't geniuses by the textbook definition, but they were still great men who changed people's lives. Why should I care if you can get a perfect score on an IQ test? What possible use is that?

    You may be a genius, but if you want to earn respect from society you've got to do more than just be.

  • Re:My suggestion (Score:5, Insightful)

    Ha! EXACTLY what I was going to post.

    In all immodesty, I'm one of those who is a lot smarter than most people in certain ways. But who the hell cares? There is more to life than being able to analyze and synthesize facts. I was a lot happier once I got over myself and figured out that there was something to learn from everyone, yes, even the point-haired idiot who wouldn't know his ass from a hole in the ground, but is brilliant in social situations. Or is brilliant at fly-fishing. Or is a great father.

    In short, everyone has strengths. It's typical of the geek to think his strengths are more important than everyone else's strengths. The biggest thing a geek can realize is that their specific set of talents is pretty damn useless for happiness.

  • by KillerCow ( 213458 ) on Saturday June 12, 2004 @11:14PM (#9410633)
    I'm incredibly intense and concentrated, yet I often become bored of specific projects in a few months.

    The thing that will determine if you suceed in life is your ability to focus for long periods of time. In school, and in life, you will have to focus on long projects to make anything. You will have to work on them after you loose the initial excitement. You will have to be able to work through frustration. You will be have to work on them after your boss / instructor / assignment makes them not fun anymore. In the real world, you will not be able to "become bored of specific projects in a few months."

    You must find something that you will love enough to be able to work through the tough times, knowing that you will be able to enjoy it again shortly, after the crunch. Your ability to suceed depends on finding something that you can enjoy -- the people that you compete with will. They will have much more stamina than you because they enjoy it so much. If you cant enjoy the lows along with the highs, you won't be able to compete.
  • Re:Own business (Score:3, Insightful)

    by Keck ( 7446 ) on Saturday June 12, 2004 @11:18PM (#9410655) Homepage
    Tell us how you really feel, cubicledrone... :)

    ..But seriously, I agree with the statement that people ought to start their own businesses more than they do; but I take issue with the larger statement that 'intelligent people are not welcome' in society.. I'm an [E|I]NTP like the author, but it's important to recognize the many different kinds of 'intelligence' just beginning to gain wider recognition. If you don't think that being socially intelligent is both valuable, and a legitimate form of intelligence, then you are probably just being self-centric.. Other people DO matter, even if they sometimes do bad things. Guess what, you do too. Not only that, but just because you are not as strong with social issues as you are with technical issues doesn't mean you shouldn't try to develop that side of your person too..

    On the whole it sounds like you've been badly abused (not physically, I hope) by the types of people you describe, in the past... Try not to stay jaded forever, if you don't make an effort to find the good in life, even among the bad, you're in for a miserable time.
  • INTP (Score:2, Insightful)

    by cfury ( 172260 ) on Saturday June 12, 2004 @11:19PM (#9410656)
    I myself am INTP, and a lot of what you said in your question applied to me when I was in school. What you really have to focus on is Work Ethics (I'm sure that's going to send shudders down a great many spines around here... :) However, what you need to do is learn to apply yourself to whatever task is at hand.

    Many of the posts I've read here have been negative. But that's just BS from the 90% of the world that doesn't know what's like to see the world the way we do. Albert Einstein was an INTP, and he wasn't understood until much later in his life (and even more so after his death.)

    Learn to concentrate on specific goals. Don't be afraid to tread on a few feet (as we are want to do.) College will help you grow and learn to understand different peoples and cultures, and there is stuff there you will learn. I highly recommend it. But be prepared for disappointment. It is highly goal oriented. Learn to embrace that.... but apply your own unique touch to it. It'll help you in the real world. But don't forget to think inside, upside, downside and outside the box. That's what true innovation is about anyway.

    Good luck!

    Chris
  • by peteforsyth ( 730130 ) on Saturday June 12, 2004 @11:20PM (#9410663) Homepage Journal
    This is an interesting point, and I'm sure it applies to many people, but I have my doubts whether it applies to the original poster.

    Another scenario is this: you have superior analytical ability, and people are generally impressed when you weigh in on a topic you've put some consideration into.

    However, as much as people are appreciative of certain specific things you have to say, they are also put off by your social awkwardness, or by their inability to figure out where you're "coming from" or what your motivations are.

    In a work environment you pursue problems that interest you, and often come up with very clever solutions, but not on the timetable others would like. You are intrigued by an interesting problem, but you are largely unconcerned with the political or self-interested motives of those around you. So you are seen as somewhat valuable, but unpredictable, and therefore not a good "ally."

    So anything that requires you to cooperate closely with others or have their trust and confidence is pretty much a lost cause. You ARE smart, you DO have talent, but much of it is wasted because people you need are wary of you.

    I believe that is also a common scenario, and my sense of the poster would put him more in that category.
  • Re:Advice (Score:1, Insightful)

    by Anonymous Coward on Saturday June 12, 2004 @11:20PM (#9410667)
    I can empathize with certain aspects of this young man's plight. I consider myself somewhat of a non-traditional learner, and I was bored to tears in school for the most part. I am not brilliant, I posses no earth shattering intelligence, but my thought processes do tend to venture outside of the norm. Sometimes this produces good results, other times it just leads to frustration when others ( who can usually find more than one person to agree with them ) fail to understand my reasoning.

    People tend to recognize my ability to "think outside the box" and often times appreciate it, but get ready to be very alone at times. In most cases others will not see things your way, and they will have plenty of other "normal" people to back them up in their views.

    Find something you love to do, and see to it. By the way, if you are as intelligent as you claim to be, you will have no need to broadcast that perception, people will notice on their own. Just keep in mind that more often than not people like you and me are no smarter than the next person, we just think differently. Now I sound like an apple ad, but it is true. Consider it more of a hardship than a blessing, you are in for an interesting ride once you hit the real world.
  • by gonzoxl5 ( 88685 ) on Saturday June 12, 2004 @11:25PM (#9410695)
    Unfortunately in the real world we often have to do jobs that we're not particularly interested in, what separates the successes from the failures is not how bright you are but how good your self discipline is and whether or not you can be relied upon to competently complete the task at hand (however personally uninteresting it may be).

    I'd hire an experienced competent safe pair of hands over a flighty whizz kid any day of the week.

    With the former I can have confidence that my project will get delivered on time and to spec, with the latter I will have to watch them like a hawk and probably end up picking up the pieces myself.

    Some people are lucky enough to be able to spend their life being creative and working to the full extent of their ability, for the rest of us the best we can hope for is reasonable job satisfaction (knowing that we made a difference) and a decent pay packet.
  • by dpotter ( 95081 ) on Saturday June 12, 2004 @11:27PM (#9410709)
    I hear this sentiment expressed far too often, and it discourages that so many people think this way. I submit to you that if your primary motivation for education is to obtain a higher salary or prove something to a prospective future employer, you've greatly undervalued that education. Pursuit of knowledge should be for YOU, not some future employer.

    Higher education represents a unique opportunity to engage with experts of all fields, to gain insight into a great diversity of disciplines and to discover where your own interests are. Your primary benefit from college is the personal enrichment you receive from education: understanding and appreciation of complexity & subtlety, insight into yourself and a desire for life-long learning.

  • Good Luck Buddy... (Score:4, Insightful)

    by jcenters ( 570494 ) on Saturday June 12, 2004 @11:30PM (#9410725) Homepage
    Hey, I've been there, I'm there, I know where you're at, so please, allow me to retort.

    First thing you should know is that there is no point to life. Wanna know the difference between Ronald Reagan and the crackhead on 8th avenue? They're both worm food right now.

    So, don't worry about crap like "success" that others foist upon you. What is success? It's whatever you say it is, nothing else. The only success is doing what you like until you too become worm food.

    Rule two: You're not going to change the world. There's too many people that have figured out how to profit from our wicked ways to let some little smart-ass cocksucker like you change that. If your goal in life is to make a lot of money, figure out how to make the heartless crooks rich and you'll be quite comfortable. Oh, and make lots of connections, because that's where the big money comes from: Connections and pure luck.

    You are a unique and beautiful snowflake. Snowflake number #3857493 to be exact.

    There are two camps in this world, the camp that says "Shut the fuck up, stop whining, eat your boss's shit on toast, put your nose to the grindstone and work yourself to death, and the camp that says "Be yourself. Let your natural talents grow. Do what you enjoy doing."

    Sorry kid. The world sucks. Everyone's out for number one and they don't give a fuck about you unless they get something good in return; no matter if it's your boss, your wife, your parents, or your kids. You're the same way, so you might as well accept it.

    And the point of my post? There is none, just like there's no point in life. Life is like an old school video game: There's no real "finish," just see how many points you can rack up until you die. Figure out your own scoring system and rack up them points boy.

  • Re:Just know this: (Score:1, Insightful)

    by Anonymous Coward on Saturday June 12, 2004 @11:33PM (#9410737)
    Sticking feathers up your butt does not make you a chicken!
  • Re:Advice (Score:1, Insightful)

    by Facekhan ( 445017 ) on Saturday June 12, 2004 @11:35PM (#9410757)
    I have always hated school too. It bears no resemblance to the real world. Don't put much stock in what they tell you. That includes the subjects, what they say about the world, and especially what they try to tell you about yourself. Like highschool its 90% bullshit. Its the same kinds of "education scientists" trying to convince you that everyone will be a more complete person after forcibly taking 30 credits of disconnected general education credits taught by TA's and part timers who probably won't even know your name by the end of the semester.

    I am fairly close to concluding that college has been reduced to just a machine to put middle class kids into heavy debt that they will have to pay off their whole lives. Why else are dorms twice as expensive as off campus living and college catering services charging 8-15 dollars for a meal. Wouldn't that stuff be run at cost if college really was about learning and not about profit?

    I have more or less relegated college to a chore at this point. I have a little less than 2 years left and I am taking some time off from it and working in my field. I am so much more successful in work environments that it is my sincere hope I won't ever have to go back. Basically college sucks IMHO. Aside from a handful of schools with extremely competitive admissions you are going to be surrounded by complete idiots. After a year you will stop wondering why everyone gets drunk, high, stoned, coked out of their mind every weekend because you will probably be doing it yourself.

    Main Advice: Go give it a try and graduate as soon as possible. Try to do something real during the summers and winters or take more classes and graduate quicker. Its a waste of time but a degree is like a vat of butter that costs 50-200k and it greases you through the employers doors. On the other hand if you, like me, don't plan on working for someone else your whole life you may want to put that money into something useful like a business.

  • Re:Advice (Score:5, Insightful)

    by deranged unix nut ( 20524 ) on Saturday June 12, 2004 @11:37PM (#9410773) Homepage
    I came to two important realizations a couple years ago.

    First, people who are intensely brilliant in one or more areas are usually intensely stupid in other areas.

    Second, brilliance does not count for anything if you don't have the discipline to stick to the task.

    You may be bright in computers and normally insightful in many technical areas, but that frequently means that you are stupid about human nature, politics, music, religion, or some other basic area. I have a co-worker who is brilliant with a debugger, but who is constantly afraid that he will be fired because he can't communicate well with his manager.

    One of my friends in college had amazing intuition. He skipped 90% of the classes one semester and consistently got A's or B's on the exams. However, when he got a job, he stopped showing up for work for two weeks, was nearly fired, and pulled the same stunt again a month later and was fired. As for myself, I would be making 10% to 15% more right now, in my 4 year old post-college career, than I currently am if I would have had the discipline to work on what I was supposed to be doing rather than what I felt like doing.

    Before you get the attitude that you are so smart, ask yourself who you are comparing yourself with. I grew up in Southern Idaho, in a town of 5000 people where I would guess that less than 1/20th of the population had a bachelors degree or better. Now I live in Seattle where most of the population has a college degree and I work for a company that employs more than twice the number of people in the county that I grew up in. In Idaho, I got the same comments that you claim to get, and in Seattle I am just one of thousands.

    Life is in constant flux. Tomorrow, psychology may be the hot field and software may be the next "automotive industry", don't choose a field because of the pay.

    Today, a college degree is frequently a prereq for consideration for any desirable job. I wouldn't skip college, mostly because of all of the extras that you learn. Life in the dorms or in a fraternity and make friends.

    Don't forget about finding someone to spend your life with, at some point computers won't mean all that much and you will want to know how to talk to someone that you are attracted to.

    If you are passionate about something and make it your job, make a point to develop a hobby or else you will burn out. Computers have been my obsession for 15+ years and software has been my carrer for the last 4 and now I play with my container garden and listen to music instead of sitting on the computer when I come home.

    Aside from that, figure out what you want to do and go do it. Life is to short to be doing something you hate. ....Most of all, take all of this with $.02....it is your life, choose for yourself.
  • Be flexible (Score:3, Insightful)

    by johnlcallaway ( 165670 ) * on Saturday June 12, 2004 @11:39PM (#9410791)
    I didn't go to college, yet my yearly income puts me into the upper middle-class range. My personal opinion is that formal college degrees are a waste of time for people that are smart and are willing to work hard. (Although I think technical colleges are vastly underrated and that for some careers, like medicine, college is unavoidable.) I go to where I am today by working hard, learning how to discuss, accept and support decisions made by others even when I disagree with them.

    Remember that no one is indespensible no matter how good you are or how much you know. Now that you know that, be prepared to take on any task your boss asks. I remember licking envelopes when I was a programmer back in the 80s because the bank needed it done.

    Second off, whatever you do, do it the best you can. A few years ago, I was loaned out as a consultant to a partner company for some vague technical skills task, but when I got there I found out all they needed was someone to sit in meetings and take minutes and publish them. Some people might have felt that it was beneath them. But if they were willing to pay my company $100/hour for my taking minutes, they were going to be the best damned minutes anyone had ever seen. After awhile, because I offered up opinions during meetings, people mentioned that I was over-qualified. But then I mentioned the above comment about the best damned minutes, and they were absolutley grateful that I was doing the task I was because the needed it done, and they were the best damned meeting minutes anyone had done. They repeatedly told my company's CIO and CEO what a great employee I was. Was I sucking up?? Maybe. But I got paid the same regardless. It was only for 3 months, and it was the most stress-free 3 months in my entire career. Besides, those CIOs and CEOs are the ones that decide who stays and goes during layoffs.

    Thirdly, don't let them take advantage of you and be honest if they try. I've been through the 80 hour work weeks and was very honest with my boss about how long I was willing to do so. He pushed it, and I pushed back, albiet very lightly. I eventually left, but it was very cordial and he called me back from time to time to ask if I needed a job. That was very handy when I was laid off several years later.

    Fourth, don't whine. If something is broke, offer up the problem and a solution to whoever is in charge. It's one thing to go to your manager and whine about the project being late, it's another to point out why it is going to be late, and what needs to be done to correct it. Whiners get ignored and become a pain in the arse.

    Lastly, when opportunities present themselves to advance, grab them even if it means shifting careers. I started out as an office clerk, but jumped at a computer operator position, then started learning COBOL for my next jump. In my current job, I am the go-to guy when no one else can fix a problem because over the last 25 years I kept current with programing languages, took sys admin, network admin, telecom admin and database admin responsibilities in different systems and learned them all. Now, I can get all these folks into a room whenever there is a finger-pointing problem and keep them there until the problem is fixed. That is very valuable; I am known around the company as the guy you can't BS and my CEO knows that not only am I the guy to get things fixed, I am also the one who can do the work if needed no matter where the problem is. Guess who will not get laid off the next round.

    Should you go to college?? If you want to, go. If you want to go into debt up to your eyeballs go ahead. There's nothing wrong with that and it might help you get a great job in 4 years. But if you are as bright as you think you are then get an entry position anywhere that offers tuition reimbursement and be the best warehouse/factory/secretary or whatever you can be. Take all the night courses you can on their dime, then watch the internal job postings and apply for anything you might be qualified for that improves your sal
  • by jelle ( 14827 ) on Saturday June 12, 2004 @11:43PM (#9410827) Homepage
    It looks to me like you first need to do some soul searching into who you really are and what you really want to do in life. Example:

    First you say "I'm incredibly intense and concentrated, yet I often become bored of specific projects in a few months." but later you say "My attention span is practically unlimited when I am interested in a topic, and I get intensely interested in it."

    And you say you love to learn, and people tell you you should be getting very good grades at school, but you don't. Do you want to learn or not? (and how/where?).

    You need to make up your mind. That means doing two things: making a choice and then setting a goal. Stop waiting what the day brings you, but take the day to where you want it to go. Take charge of your own life.

    You seem interested in many things, but keep getting worried that you're missing something better if you stick with it, hence the feeling of boredom sets in. Independent of whether you actually do have attention problems and can't stick with something, or possibly you do have a practically unlimited attention span (but you are still not sure what for) what you need to do is look farther into the future and set an ambitious goal. Imagine yourself 10 years from now, and what it would take for you to see yourself being happy and successful. I'm saying you and yourself and I mean it, don't go for the bland 'commonly accepted definitions' of happyness and success, but what really is it that makes you happy and what would you consider a success when you look at yourself 10 years from now. For some people that means having a family of your own (loving wife & kids), for others a particular career (money, respect, power), for others a particular social position in society (love, respect, power), and for some it is linked to a geographical place, or other people, or a particular surrounding, also religion may be a factor, etc.

    It can help to add the 10 years to your age and search for people in that age group that can (partly) serve as a role model or guide. Your personal role models don't have to be alive, or currently in the target age group, but it can be very helpful to research 'what did eeeee do when he/she was that age'.

    When you know where and what you want to be in the future, that will tell you exactly what steps to take now and will help you make all those smaller choices needed to get there.

    Just my 2cts worth...

    Myself I feel like I'm just in the process of achieving current long-term goals and I must say that I am happy and feel successful, and now I am searching for a new goal. I am confident I will have a much clearer picture of it by the end of this year, and for you: I hope you do too.

  • Re:Advice (Score:2, Insightful)

    by SpootFinallyRegister ( 787720 ) on Saturday June 12, 2004 @11:52PM (#9410884)
    does that sound harsh? maybe, but it hits the nail on the head.

    I used to feel the exact same way. Its probably why I dropped out of college and only recently went back and graduated (at 26). These are the kind of things I needed to realize before I could pull my head out my rear, get off my pedestal, and actually accomplish anything.

    Yes, we are bright. But just because Einstein was disinterested too doesnt make us geniuses and wont make us successful. Even if it did, sitting around and contemplating how wonderfully non-traditionally we are is an excellent way to severly underachieve.

    We are not non-traditional. In fact, I can't count the number of people who feel exactly the same way about themselves, yet I don't know anyone who is good at everything. Leonardo da Vinci was non-traditionally bright. Being good at math and average at philosophy does not make me a special unique snowflake, and certainly shouldnt excuse me from philosophy.

    Just because something doesnt interest you doesnt mean you dont have to do it. Taxes dont interest me, reviewing co-workers bugfixes dont necessarily interest me, and figuring out a 401(k) doesnt interest me. Good luck not doing any of them. Life is full of boring, tedious crap you have to do, no matter how great and interesting a job, project, thesis, or whatever you stumble upon. Hopefully most of what you do is interesting, but it will never all be intetesting. Learn to get yourself through the boring stuff now -- it will always be there, and the more ingrained you get it in your head that youre above doing them, the more it will handicap you. Whether thats writing a history paper or endlessly bouncing around design documents in a large company, it happens.

    Heres the most important part... really think about this and be honest with yourself. For those things that dont interest you, is it possible that you arent doing them simply because you arent good at them? that facing up to the fact they take you more time than other things (sometimes even *gasp* as long as it takes other people to do them) is too difficult, and its better to do poorly and blame it on disinterest? it will probably take you a few years to answer that one honestly. when you can, itll do wonders for you.

    Talents are a gift, but you have to ascribe value to them yourself. People wouldnt think too highly of Thomas Edison if he was too lazy to apply himself and ended up shoveling manure out of a barn.

    Things will change quickly when you go to school. Thinking you are the smartest, you are different, and that you should get special consideration is a crock of shit. Realizing that is a step you have to make to go from being gifted to being smart.

    Watch out that you arent just finding a nice way to candy coat laziness.

    Good luck.
  • by tentimestwenty ( 693290 ) on Saturday June 12, 2004 @11:57PM (#9410921)
    You're probably going to hate college/university if you've hated school up to this point. Most people/programs at the collegiate level are pretty dumb and focus strongly on learning by rote, passing tests and writing cookie-cutter essays. If you really are bright you won't be challenged by 99% of the courses offered- best just to read some books. If you're intent on going (and you pretty much have to unless you're also a good entrepreneur) my advice is to first take courses that you wouldn't rationally want to take. Take drama, science of sound, taxidermy or whatever you can that has a twist to it. You're likely to find similarly bright people in these courses. Some of them will be plain geniuses in a variety of weird ways. I took visual art which is really one of the only explorative disciplines. I was lucky to have some exceptionally gifted teachers and our program was heavily philosophically based. In short, it was a bunch of crazy creative geniuses who were all learning about "meta" things. It was probably the only thing that saved me from complete despair after high school. The people I met were challenging and intense and the environment was rich for arbitrary boundary definitions. The courses at times were mind-blowing, but in the best way, there was always something coming next that was better, and always different.

    As others have pointed out, this kind of nurturing and challenging environment goes a long way to "getting over yourself" and onto the path of doing something worthwhile with your brightness. In the end, if you believe you're bright you have to test your character by doing something great, failing repeatedly, and finally finding your true path. Unless you stick yourself in the midst of people at least as bright as you (and why not go for the cream of the crop?) you will forever hold yourself in some false and paralysing image.
  • by daviddennis ( 10926 ) * <david@amazing.com> on Saturday June 12, 2004 @11:59PM (#9410935) Homepage
    I think a high percentage of the people here are pretty smart, because Slashdot really doesn't have much for people who are not. That being said, since we're debater-types, we tend to be a little mean-spirited. I apologise on behalf of my fellow Slashdot users for the insulting tone of many of these messages.

    The fellow who edited your comment mentioned that you were probably an INTP. This is true; so am I. This means that you are devoted to finding logical solutions to problems, and are dreamy and absent-minded if you're not involved in something that interests you. This would seem to fit your educational profile to a "T".

    About 1% of the population are INTPs. Since they're logical and like designing things, they tend to gravitate towards computing as a career, so you see a very high percentage of them here.

    Perhaps the most revealing thing about the Meyers-Briggs type indicator, which is where these strange four-letter acronyms come from, is that people are very different, and many of the differences can be described by a simple formula. I've found that even with very complex people, the Meyers-Briggs attributes make it easier to deal with them and understand at least parts of how their minds work.

    A good example of how people think is based on logic. When I was younger, I thought logic was the be-all and end-all, and that it was simply impossible to make sense of contradictions. Now I understand that there are people who don't care about contradition; they just care about getting work done and if this means doing things that are not strictly logical, well, that's what will be done and that's what they need. This is very important to understand when programming systems such as reports which may have seemingly contradictory attributes. A pure INTP would simply say its not possible to do them. An INTP with some seasoning and social understanding will try very hard to untangle the contradictions and find a solution that works.

    Many times the best type of person for you is someone very different from you. People who use feelings to make decisions, for example, are capable of deep love and can make wonderful relationships. People who are strictly logical wind up looking cold and characterless, both to that type of person and to each other. So if you check out the Meyers-Briggs and use it to classify people, don't forget the feelers. They may bring some much-needed passion into your life.

    Now, it's worth noting that types are not the be-all and end-all. They don't describe everything about a person. I have dated a couple of INFJs, and they've always been special to me. It's clear to me that I have a real affinity to that type of person. But both of them were very different and distinct people, despite having similar basic personalities. The one I'm involved with now is a wonderful creative artist who has brought much joy into my life.

    I've used these four-letter acronyms so much I feel like i should explain the MBTI a little. Full knowledge of it takes whole books, but at the root, it's simple. There are four different attributes that define a personality in the MBTI:

    Introvert/Extrovert (I/E). Are you energised by being with other people, or by being alone?

    iNtuitive/Sending (N/S). Do you concentrate on things as they are (sensing) or as they should be (Intuition)? Do you think of things as concrete facts (Sensing) or Principles (Ntuitive)? As an iNtuitive person, I get along much better with my fellow dreamers than with those bores who are sunk in drab reality.

    Thinking/Feeling (T/F). Do you make decisions based on objective fact (Thinking) or by the effects they have on others (Feeling)? Most people in the computer field are thinkers. A large percentage of women are feelers. This is why computing is such a male-dominated field, and why computer people tend not to have a good understanding of the opposite sex.

    Perceiving/Judging. Do you have a clean desk (Judging) or a messy desk (Perceiving)? Do you pre
  • by Anonymous Coward on Sunday June 13, 2004 @12:05AM (#9410961)
    If I were to come up with a list of books geeks should read, "How to win friends and influence people" would be right on top of the list. It could be the most important book many young, angry slashdotters, who feel alone and isolated, could read. The main principle is simple: People like people who are interested in them; people don't like people who are overtly interested in themselves - describing yourself as "uniquely bright" is one sure-fire way to annoy people. Someone should do a slashdot book review.

    As a corollary, I'd quote another poster "FIND AND CHERISH GOOD FRIENDS." What you can't do alone, you can do with other people.

    Second, be a pragmatist, not an ideologue. Someone once said politics is the art of the possible, the art of compromise. Pick up a biography of any great political figure, and you would read of compromises he had to make. What works for world leaders works for ordinary folks - to get somewhere, you need to compromise, do things that aren't immediately exciting or pleasant. If you're really "uniquely bright", you should be able to figure that out.

    On the other hand, don't try to please everybody. No one who was liked by everybody ever got anything significant done. This seems to contradict my points above, but again, it's all about compromise and balance.
  • Re:Advice (Score:2, Insightful)

    by John Jorsett ( 171560 ) on Sunday June 13, 2004 @12:12AM (#9411007)
    I never saw the point of learning French, the effort required was vastly disproportionate to the benefit, and I have lived in the country for two years.

    Ah. I was wondering about the gratuitous Bush bashing in an otherwise on-point post. You may not speak French, but propinquity has given you some of their attitudes.

  • by UserChrisCanter4 ( 464072 ) * on Sunday June 13, 2004 @12:14AM (#9411016)
    Is two-fold.

    1) As some others have pointed out, you're basically a kid (although possibly legal) who graduated from high school. Bright though you may claim to be, you haven't really demonstrated anything in terms of practical intelligence. Get off the high horse and prepare for step two.

    2) Sit down with your folks and talk this through with them. Spend a year living on your own in the real world. Trust me, it's only gonna take a year. Move out, get an apartment, pay some bills, get a credit card and learn to use it correctly (or better still, royally screw up and be thankful that you're only 18). You don't have to go all out and get a car loan (if you can avoid it, because it's going to keep you tied into this lifestyle, so try to get a car from the folks), but avoid living at home during this experiment at all costs.

    Try to obtain and hold down two or three of the following jobs during the year: Retail Sales/clerk, some sort of receptionist/secretarial/clerical work, car sales or some similar "high stakes" sales job, or some sort of construction or low end mechanic work (a jiffy lube or similar). These are the sorts of jobs that a person without a degree can work in and, to a point, actually sustain themselves. While you can hear stories all day long about guys who have sys-admin jobs with no degree to back them up, the fact is that those days are pretty much gone, and there's enough guys out there with a CS degree who will work the same job that it'll keep you at "Mel's Used Cars" indefinitely. The up-side to these jobs is you'll learn some cool stuff that will have a practical application in your life later on. You can pick up some good info on how car dealerships work, and how to keep from getting scammed. You can play retail from the other side of the counter, and chances are that you'll be kinder to retail clerks for the rest of your life. Knowing basic construction skills will save you huge amounts further down the road when you own a house and don't have to pay somebody $1,000 to hang some sheetrock in that room over the garage you want to turn into a LAN lair.

    In about 4 months it's going to dawn on you that things like the basic food in your house cost a fair chunk of cash, that car insurance is ludicrously expensive, that landlords aren't always the best people but work well with give-and-take situations, and that living with a roomie isn't always the hilarious life sit-coms make it out to be. You're going to start to realize the amount of money it would take to live and be self-sufficient, and the amount of money it will take to do anything other than "tread water". When you hit December or so, apply to the university or community college of your choice, because come May you're going to be sick of this "real world" crap, but more importantly, you're going to realize that although 50% of college is bullshit classes and random facts that you'll never need to know (I can tell you that the word 'file' came into the English language through middle French, and is named for the thin string originally used to organize 'files' in a cabinet), but part of the point is proving to an employer that you can slog through bullshit. People will change careers, on average, five times in their life. Get a degree in a subject you enjoy, even if it's History or English, and try to study some interesting subjects in your electives. Your first job may not be exciting or pay mad Benjamins, but by this point you'll have already figured out that work isn't fun time. It shouldn't be crap, mind you, and with luck you'll also have learned how to spot crap employers, but you'll be a little more understanding of how life actually works, and you'll realize that work isn't supposed to be demoralizing, but it isn't usually fun either.

    When you get a real job, one where you have weekends and two weeks of vacation, you'll have time to pursue your weird side interests and linux and tinkering and everything else you adore. Not oodles, no, but it'll be there. Try to keep yourself reaso
  • by bcs_metacon.ca ( 656767 ) on Sunday June 13, 2004 @12:16AM (#9411035)
    "You are not your job. You're not how much money you have in the bank. You're not the car you drive. You're not the contents of your wallet. You're not your fuckin' khakis. You are the all-singing, all-dancing crap of the world." ...as you grow older, you'll realize you're not special, you're just smart, like a lot of other people. And you'll meet people a heck of a lot smarter than you. Smarts is not as important as wisdom, and neither is as important as a sense of perspective.
  • by jcenters ( 570494 ) on Sunday June 13, 2004 @12:24AM (#9411084) Homepage
    You're a smart guy Spara. That's where we're cursed.

    See, what most people don't know is that smart people like us, like many on Slashdot, are cursed. Intelligence is a frigging prison.

    Why? Because we think ourselves out of the white picket fence. I live in the south where most rednecks are happy working in a factory and knocking up women like it was going out of style. And they love that lifestyle and God bless em for it.

    But us? We think too damn much. We think until we push ourselves outside of the Matrix(TM) , and we sit here, eating some snotty goop and thinking "Why?'

    So all you dumbasses out there, you know, those that could never fabricate a circuit board, never write a line of Perl, or tell the difference between Everclear and a nice Amarone: Consider yourselves lucky. You live nice, happy, simple lives. Fulfilling lives. You feel good when you die.

    Us, the "smart" ones, on the other hand, live like savage refugees. We ask too many damn questions and pay the price for trying to pick the brain of God.

    Oh, and before I go too off-topic: In Soviet Russia, Natalie Portman's hot grits outsmart YOU, All your Base are belong to step 3, Bill Gates is the sux0r.
  • by poloace ( 315435 ) on Sunday June 13, 2004 @12:28AM (#9411111)
    I have a similar background and here are some things that I've learned over the years:

    - College wasn't for me. I dropped out and fortunately it worked out ok. Don't be afraid to take the old school, traditional path (on the job training).
    - You won't have patience for routine things like paperwork, paying bills, etc. You'd better develop some serious self dicipline or you'll be screwed in the long run. (suggestion: join the military to develop those skills well)
    - You'll think you know more than everyone else. There are valuable lessons to learn all around you every day. Develop good relationships with highly successful people. Watch them closely and learn from them. You'll be surprised just how much more there is to learn, especially in fields outside your own.
    - Having the biggest brain is not the most valuable item to most companies. Having diverse expert knowledge and sharing it in a constructive way with everyone around you is highly valuable to companies. You end up becoming a force multiplier to the company. Everyone around you improves. If you don't develop your social skills though and use them in a way that people can accept, you'll poison everyone against you. People skills count in a big way.
    - Most jobs have some level of routine tasks that have to be performed on a semi-regular basis. Mix a little interesting work into each day to keep yourself from getting bored. Don't let it dominate to the point of not getting your key work done though.
    - Don't have contempt for the others around you. If you see them as people with little knowledge compared to yourself, then you aren't really seeing the complete person. You're only comparing them against your own skillset. You need to look for unique qualities and skills that they have that you may not. Appreciate them for their own gifts and talents. If you can see where they're better than you in other areas, you'll not approach them with arrogance and contempt. By understanding those unique talents, as a team you'll all be more effective.
    - Take on the areas or problems that no one else wants to take. You'll find that there's usually a hidden jewel in that area and an opportunity to show your creative talents. In doing so, you'll provide great value to the company.
    - The suggestion of reading "How to Win Friends and Influence People" is right on target. I live by those principlines and it's served me very well in business and my personal life.
    - Take everyone's advice constructively about getting over yourself. You may be right about the fact that you're bright, but trust me, there's always someone brighter. If you're lucky, you'll find someone like that in your career. They'll usually put you in your place a bit. If you find someone like that that also has strong people skills, try to get them to mentor you. I had someone like that and I owe him a lot in my professional development.

    Good luck
  • well... (Score:2, Insightful)

    by dayhox ( 696422 ) on Sunday June 13, 2004 @12:29AM (#9411115)
    After having had a word with my old boss, a retird Lt. Col U.S.A.F, regarding my wanting to take a break from college, he said earnestly "You know, the biggest mistake intelligent people make is their belief that the rules don't apply to them." I didn't listen to him then, and after 6 years, am still on the break.
  • by dtperik ( 695891 ) on Sunday June 13, 2004 @12:32AM (#9411133) Homepage
    Executive Summary: The more you know, the more you know you don't know.
  • by Shajenko42 ( 627901 ) on Sunday June 13, 2004 @12:35AM (#9411144)
    It will also teach you that your ability to manipulate other people is more important than your ability to do anything else.
  • by burtonator ( 70115 ) on Sunday June 13, 2004 @12:35AM (#9411147)
    Seriously... leave now. College will just suck your soul out of your body and leave you just a dry husk ;)

    I did it... went one year. My career hasn't been slowed down at all in fact it's taking off.

    My company has funding, I'm at peek form, really know my shit, have some amount of respect in the industry, and college has never been an issue.

    Anyway... Rock on! :)

    PS. College is really good for some people... If you have ADD it's a good sign to quit. The system isn't designed for you...
  • Philosophy! (Score:4, Insightful)

    by lux55 ( 532736 ) on Sunday June 13, 2004 @12:40AM (#9411168) Homepage Journal
    There's so much to learn out there, much of which is unfortunately not accessible via the modern university (at least not the main paths through it), that would both provide a good mental challenge for you, as well as help channel your intellectual gifts into positive outlets.

    The classics and the medievals especially, btw. While the moderns are important, I'd argue that that's so because we need to figure out where we went wrong in order to right ourselves. The classics, on the other hand, and the medieval philosophers especially, had such a huge emphasis on method, which is so critical to making any progress at all (methods beyond the impirical scientific method).

    The empirical evidence to the importance of philosophy, in case you're skeptical of it, is that most of the great scientific and mathematical minds throughout history considered themselves philosophers, not scientists, and considered the two inseperable. The major artists too, you'll find, also considered themselves not artists but philosophers.

    PS. Philosophy is Greek for "Lover of Learning" or "Lover of Wisdom". Can't top that if your hunger for learning really is insatiable. ;)
  • Re:Advice (Score:5, Insightful)

    by the_ed_dawg ( 596318 ) on Sunday June 13, 2004 @12:54AM (#9411223) Journal
    The real thing you need to do is get over yourself. You're not special. There's lots of people in this world that are just as smart as you.
    I'm normally a very encouraging person, but I have to agree with the parent here. This leads into something my mother would always say, "There's a difference between being smart and wise." I know a guy who was a supposed genius. He entered college on a full scholarship despite getting a 2.0 GPA in high school. He was way to smart for his electrical engineering classes, so he never went and stayed up late doing stupid things like riding the glass elevator at the Hilton and strolling around Wal-Mart looking for blue lamps. In the end, he lost his scholarship after a single semester (failing to keep a 2.0) and joined the Air Force, where I have heard he became a real screw-up.

    The first question everyone asked was, "He was so smart, so why didn't he succeed?" The answer is so obvious that few people were willing to see it. He wasn't wise enough to know that he wasn't as smart as he thought he was. The second reason is that very few people give a damn about how smart you are. You will be judged based upon what you complete, and if your attention span is limited only to what you find interesting, you are destined for failure. Your boss will ask you to do something that is not interesting to you because it needs to be done. If you make a habit of (a) saying "no" or (b) not doing it with the effort required for successful completion, you can complain to the people standing next to you at the unemployment office.

    Reiterating what many others have said, stop whining and prove your worth. I'd recommend college because I know very few employers who will even consider hiring people without college degrees for anything less than factory assembly or janitorial staff. Why? College doesn't prove that you're smart because lots of stupid people graduate each year. College shows that you can dedicate yourself to a long-term goal of accumulating a functional knowledge of a discipline and succeed at an acceptable level. If you aren't willing to do that, be prepared for heartbreak, as most companies are not willing to take a chance with you, however promising you claim to be.

  • by Aczlan ( 636310 ) on Sunday June 13, 2004 @01:06AM (#9411274)
    "Just because someone doesn't know anything and everything about computers and networks doesn't make them an idiot, it just means that they actually have a life. You oughta give it a try, although it's probably too late."
  • by dbullock ( 32532 ) on Sunday June 13, 2004 @01:23AM (#9411348) Homepage
    Slashdot is not just for those who are pretty smart. It's primarily for those who want to bathe in the group meme, patting each other on the back repeating the same tired chestnuts.

    I read slashdot with my filters cranked up high only because they do a decent job collecting often interesting news. The comments with exception of a small few are usually pretty worthless retreads.

    Slashdot is for the most part, the ultimate pickmeup for the members of the herd when the individual members feel insecure, and need a pickmeup that's only a few clicks away.
  • Re:Advice (Score:4, Insightful)

    by Amiga Lover ( 708890 ) on Sunday June 13, 2004 @01:30AM (#9411373)
    The real thing you need to do is get over yourself. You're not special. There's lots of people in this world that are just as smart as you. Once you get over yourself, the world is your oyster. "unusually but non-traditionally 'bright' "...jesus...Kill me. Get over yourself.


    I'd mod you up if I could, but you're already at +5.

    Something that's always annoyed me about geeks (the computer kind) is how much crap they'll spew forth about being special, brighter, smarter. These are people who can work well in a few areas, generally a little academic, and often loners. Go within a few degrees of that description and you have most geeks, most of the slashdot crowd.

    Now, not to take away from that - it is a good thing to have capabilities, but so often I see this same type expect that by virtue of that unique set of talents they are above many others. That's so many levels of bullshit. I've known carpenters who can't turn a computer on and barely read who can push out some amazing work from nothing but 3 tools and a couple of boards. Gardeners who can build phenomenal areas with a serenity and comfort matched by nothing else. Musicians who would be lucky to see ten feet in front of them for being lost in their own world, but who can pick up a single instrument and play to melt your heart.

    We're nothing special. I'm a geek myself, and that's my specific talent but I don't in any way think that makes me especially bright, or uniquely so in any way other than being a unique individual.

    And we all have that.
  • Re:Advice (Score:5, Insightful)

    by Ohreally_factor ( 593551 ) on Sunday June 13, 2004 @01:59AM (#9411489) Journal
    I disagree. He is special and unique, just like everyone else.

    My advice:
    I'm not entirely sure what the big rush is to go to college. I recommend living in a foreign country first, preferably one with a different language and a different culture than your own. Find a university or college town or smaller city in this country and plop yourself down for a year to learn the language. Avoid places with huge tourist industries, but if there is a little tourism, that's good. It will give you an opportunity to meet other travelers and the occassional opportunity to hang out with a countryman (or woman). Also, there's a better chance other people will be able to speak your language.

    Now, if you don't have much money to start with, let me suggest a third world country, or at least a developing nation. Work for 6 months, save $300/month, and you'll have enough to live in Mexico or Central America for a year, if you live very frugally. I know students that are renting decent rooms for $75 US in Guanajuato. You can eat pretty well for $100/month, less if you cook your own.

    Once you pick up enough of the language, you can look for opportunities to supplement your income. Do you want to take language classes? It might be possible to teach an English class in exchange for a language class.

    Travel really does broaden the horizons. Real travel, that is. A two week vacation is generally not long enough to really soak up a different culture.

    At the end of the year, evaluate. What do you want to do next? More travel? College? Work? Some combination of the three?

    The most important thing is not to get sucked up into the values and expectations of those that haven't really examined life. They'll tell you everything you need to know about being successful*, but they know nothing about being you. If you just focus on being you, or as the parent poster suggested, being the best you, you'll do much better than 99% of the other unique and special people.

    *I advise going into plastics.
  • by Anonymous Coward on Sunday June 13, 2004 @02:11AM (#9411527)
    Congratulations on finding your calling. The world definitely needs teachers that can inspire and help students to achieve...
    Unfortunately society and the students themselves don't always want what they need.

    So just remember, when in 5 or 10 years time, or whenever you get to the point where you wonder why you got into it, after the 15th kid that day swears at you, does the fingers and walks out saying "his dad reckons only losers have to teach", that it wasn't a mistake.

    Some kids can't be helped. A lot of kids just don't care. But some out there are just waiting for someone to give a damn, and given a great teacher, and a good mentor, will go on to do things that will help change the world for the better.

    Good luck.
  • Some more advice (Score:3, Insightful)

    by ArsSineArtificio ( 150115 ) on Sunday June 13, 2004 @02:21AM (#9411577) Homepage
    And the point of my post? There is none, just like there's no point in life. Life is like an old school video game: There's no real "finish," just see how many points you can rack up until you die.

    Rule Three: get religion, or you'll end up a bitter, twisted weirdo like this guy.

  • by Tripster ( 23407 ) on Sunday June 13, 2004 @02:27AM (#9411597) Homepage
    Ok you two, you've now figured out the truth and will have to be taken out. :-)

    I've pretty much concluded the same thing, life is actually pointless in that it is only a temporary thing anyway. As it sits we could be wiped out of existence tomorrow in many ways, at which point everything we've ever been is gone and was entirely meaningless.

    So, do what you enjoy doing, it will all be over quicker than you'd like anyway more than likely, I don't imagine too many folks are ready for death once it comes, although I expect they don't realize that after they are gone either.

    You only get one ride, enjoy it while you can because it's gonna be dark a long time afterwards. Life is basically a vacation from death some author once said.
  • by Anonymous Coward on Sunday June 13, 2004 @03:16AM (#9411728)
    First, the cynical stuff. Yeah, you may be in the top 1% of the nation, but with a population of 250 MILLION that still leaves you with 2.5 million peers. You're as special as any individual, but you aren't THAT special. Just one of those sobering facts of life.

    And damn the self esteem movement for foisting the "you're spai-shul!" mentality on us*shakes fist*You aren't entitled to anything. You aren't God's gift to mankind. You just are. Self assurance is important for quality of life, but dont let it become hubris. Don't take an antagonistic approach with the world or it WILL smack you down. It'll probably smack you down anyway, but if you can separate self worth from external factors and understand the world doesn't "owe" you anything or even care and that doesn't mean dick, you'll be better equipped to roll with the punches. You'll have the perspective to see set backs are just that - set backs - and don't mean anything more than "I need to try again or try a different approach." Which SHOULD have been the message of the self esteem movement, but it somehow got lost in feel good coddling of wounded egos.</rant>

    Secondly, a lot of the comments here are spot on. Raw ability counts far less in this world than discipline and persistence. Talent can take you so far, but to be truly successful or stand out it also takes a lot of hard work and a little luck. Just as important is your ability to relate to and work with other people. The days of the lone genius toiling in solitude to make a breakthrough are all but over. These days, just about any field you end up working in is going to require you to be a part of a team. Also, sad but true, advancement in the academic, corporate AND governmental worlds is going to be determined just as much by how well you relate to your superiors as it is by your raw performance. If you cant maintain good relationships with others, expect to be frequently passed up for less qualified candidates.

    Maybe instead of college you should consider working a crap job full time for a while - for instance, call center work. In my case, THAT more than anything gave me to willpower to get through the boring, mundane crap that awaits in college. It also taught me how the 'real world' works and that: A) it isn't as bad as you think and B) you can be the brightest person in the world, but it wont be worth crap if you don't produce results and cant gird yourself to put up with the politics and other 'mundane people stuff' that make the world tick. Oh, and C) it taught me that I don't ever EVER want to be stuck with that kind of work again - therefor, no matter how boring, I will put up with all the BS so I can get to something better.

    All that said, a lot of this just sounds like adolescance. There's certain wisdom to the old idea that some people are just late bloomers. This goes mentally as well as physically. Modern imaging techniques have taught us that changes in brain structure continue well into your twenties. Your lack of focus is in part just a sign that you aren't done growing yet. Give it time, and try avoiding college for a while so you can work on other skills. Saving money from a job also doesn't hurt. And you'll do a lot less damage to yourself if you fuck around and switch work frequently than if you go to school and get a bunch of F's on your record. Not to mention you wont be throwing money down the toilet for the privilege.

    Yes, I really this topic is probably a troll and I realize it's been beaten to death, but I wanted to throw my inflation adjusted $.02 in anyway.

  • by Leebert ( 1694 ) on Sunday June 13, 2004 @03:43AM (#9411802)
    Try to obtain and hold down two or three of the following jobs during the year: Retail Sales/clerk, some sort of receptionist/secretarial/clerical work, car sales or some similar "high stakes" sales job, or some sort of construction or low end mechanic work (a jiffy lube or similar).

    I can't agree with this more. I spent a year in construction prior to my current job. I had quit my previous job to take a new job. Unfortunately, my new job was contingent on me passing a criminal background check, which I did not. (Long story ;) ) A friend offered me a job hauling around drywall supplies to job sites. I considered it so beneath me, but I always believed that an able-bodied man had no excuse not to work, so I took the job.

    Several weeks into the job, I met a finisher named Carl. Carl had a masters in EE. Carl found his old job too stressful and bothersome. He quit his old job and started finishing drywall. He made more money doing that ($xxx,xxx), and he was in a quiet, stree-free job. Carl spent a good bit of time helping me with some Calc II problems I had.

    After meeting Carl, I started to look more closely at the folks I was working with. I began to realize that just because a guy was wearing muddy boots, it was not necessarily an indication that he was an uninteligent person who didn't go to college. No, many of these folks had degrees, were intelligent, made more money than my then-dot-com friends, worked reasonable hours, and immensely enjoyed their jobs. I realized what an arrogant snob I had been. I realized what a high degree of prejudice I was exibiting. Wow, what an eye-opener.

    I started to realize how much I was enjoying the job. I drove around a truck all day listening to the radio (NPR, C-SPAN, and whatnot), enjoying the outdoors, and getting excercise. Lifting drywall isn't easy, but it's good for you physically.

    When I watched Office Space a few years later, I identified with Peter at the end very well. It was nice to get out of the office for a little while. I gained some skills that I can use in life. And I gained substantial life lessons from the experience. These days I work in high performance computing at NASA, which is a stark contrast. But even there I apply a lot of the life lessons I learned driving the drywall truck.
  • Re:Advice (Score:5, Insightful)

    by Anonymous Coward on Sunday June 13, 2004 @04:00AM (#9411841)
    >> I have always hated school too. It bears no resemblance to the real world ... I have a little less than 2 years left

    It's wonderful that only 2 years into college, you've got the real world all figured out. In reality, you haven't reached the best part of an undergraduate education yet, and you certainly haven't gotten an education in the real world.

    >> Like highschool its 90% bullshit.

    That's code for:

    "I think I'm really smart, but I haven't done that well academically. Therefore, school must have been bullshit."

    Sure. Basic math, science, and writing skills are a complete waste of time, as are history and the social sciences. Right. You were ready for the real world right out of middle school!

    Back to reality again. The general education classes will teach you a LOT about a wide range of things if you're willing to put a little effort into them.

    Ah, what do I care? Feel free to go through life a complete ignoramus when it comes to subjects that don't interest your narrow, closed-off mind.
  • by StarKruzr ( 74642 ) on Sunday June 13, 2004 @04:01AM (#9411845) Journal
    ... but shut up.

    There are *MANY* reasons to go to college other than gaining an education in a chosen field. I made friends in college that I will know for the rest of my life, and grew incalculably as a person.

    And no, you can't get the same experience working at a company somewhere.
  • Re:Advice (Score:3, Insightful)

    by spectral ( 158121 ) on Sunday June 13, 2004 @04:07AM (#9411865)
    I realize that it's expensive and nearly impossible, but it seems to me that people that aren't challenged in school and find it boring, should be taught differently. Some people pick stuff up rather quicker than others, myself included. Do I think this is a good trait? yes. Did it lead me to being bored in high school (and college, since I couldn't afford a 'better' one) most of the time? yes.

    It seems to me that were I to have been challenged, I might have developed a work ethic. I'm not saying I'm the smartest person ever. I'm not attempting to gloat. I'm just saying that the current educational system is trash. I was bored senseless in high school. I actually slept through most of my classes, and still got A's, because I did the minimum amount of effort required of me to do so, which wasn't much. Now I'm looking for life in the real world and I have no work ethic. Were I to be challenged, I'd probably just give up. I never learned to 'study' for the classes that I needed to, and just ignored them. I didn't fail them, and still graduated with honors, but it was annoying because I didn't get most of what I could from them.

    Any time where I can attend, either physically or just mentally (i.e. not daydreaming), any class only 6 times and still get an A is poorly structured for someone such as myself. What do I recommend? Well, one on one education with someone I consider an intellectual and knowledgeable equal on the topic of discussion would have been amazing. But how many people can afford that?

    I don't have a solution, but it seems to me that if any structure set up for education is boring to you, then you're either in the wrong field, or it's being taught wrong, at least for the way you learn/think.

    Too many times I've sat in a class and not paid attention for 30+ minutes while the teacher/professor explained something that seemed easy enough (took about 5 minutes), then spent the next 25 re-explaining it to one or two people in the class who just didn't get it. No offense to them, but I'd prefer that clarification such as that should be done on their time, not everyone's time.

    Mod me down for being a pretentious asshole now, it's rather hard to talk about how you're better in some way than the majority of the population (and frustrated by it) without sounding like one. I really don't think I'm better than anyone else for it, it's just one of my gifts. Others can sing, do artistic things, whatever. Mine was quick understanding of various logical topics/pursuits (math, computers, mechanical/technical drawing.. anything seemingly with set rules instead of arbitrary decisions, though admittedly programming is more of an art than anything else, it's a logical art. Though, most could say that art itself is better thought of in terms of rules and logic as well :))
  • by dubl-u ( 51156 ) * <2523987012&pota,to> on Sunday June 13, 2004 @04:27AM (#9411914)
    you don't have to take stupid, boring, and irrelevant to your interests classes

    I'm sure there must be people whose interests at 18 are perfectly tuned to match what they'll need to know over the next 30-50 years. But I don't think I've ever met one.

    When I was just starting college, I didn't take much in the way of advice, so I'm not sure it will do any good to offer it. But just in case, here's my take:

    I would have been better off in college had I done the exact opposite of my instincts on a regular basis. Instead of following my muse all the time, I would have come out better had I picked one thing and stuck with it, come hell or high water. But because I could get away with hop-scotching across the departments, I reinforced the same bad habits that I had developed in high school, the same bad habits that many bright kids develop. And I avoided learning a lot of things that would have served me well later.

    One tip I can offer is that you should work assiduously at knowing and challenging yourself. Through careful observation and experimentation, learn exactly why and how some things are hard for you, and some easy. Then constantly push those limits. Focus especially on cutting through your own bullshit; smart people are often incredibly good at fooling themselves.

    The other is to do nothing half-assedly. Before you commit to college, figure out exactly what it will take for you personally to come through it with flying colors. Then when you commit, really commit. Or if you aren't sure you can do it, then find some easier stepping stone (like a 1-year program, an apprenticeship, or even a single college class) and learn how to play and win that game before proceeding to the big leagues.

    Another way to develop the character necessary to tackle college is to try getting by without it. As soon as you're done with high school, get a job and move out of your mom's house. You will soon discover that what matters is not your raw talent, but what you manage to accomplish with it.

    None of this should dissuade anybody from nurturing their muse; their are more important things in life than churning out widgets. But I know a lot of bright people who did fuck-all with their talents, doing nothing but dreaming their lives away.
  • by dankjones ( 192476 ) on Sunday June 13, 2004 @04:58AM (#9411979) Homepage
    Your weakness is that you underestimate people too easily.

    You are unhappy because you've just recently realized that alot of the world sucks, but have yet to realize just how lucky you are, learn to give a crap about people other than yourself or become aware of any worlds outside of your own tiny little reality.

    Most people are smarter than YOU think.
  • Re:Advice (Score:4, Insightful)

    by Mycroft_VIII ( 572950 ) on Sunday June 13, 2004 @05:00AM (#9411985) Journal
    I hate to burst your bubble, but #1 is only true with idiot savants and a few other cases. You've bought into a stereotype. And an easy one to buy into as most want to believe in some sort of cosmic fairness. The other reason is a lot of smart people simply don't place priorities the same way as others and get considered clueless for not playing keep up with the Joneses or choose a happy life instead of an ambitious one.

    Mycroft
  • by Anonymous Coward on Sunday June 13, 2004 @05:05AM (#9411998)
    After reading your parent post I had the same, "That's just like I felt at that age" feeling that many /.'ers have posted.

    I was considered bright at school top of the school in GCSE's (the UK 16 year old exams), most academic awards given to one student at the school, etc, etc, etc. I had meetings with two teachers every month to make sure I did not burn out. I was even helping teachers out with lesson plans (there are a biology and a physics lesson that I wrote that the school are still using to teach year 10's with).

    I have always imtimidated people I converse with and consequently have very few friends. This does not matter to me as most people are full of crap anyway. However, there will be a few people you meet (and you will probably meet these people at University) that you will value the opinion of. These people will become your life long friends. By friends I don't mean people you meet down the pub, I mean people who help you move house, people who will tell you your current girlfriend is the best/worse thing in your life, people who tell you what you need to hear even if they know you may not talk to them for a whole year afterwards (because it sometimes takes you that long to work out they were right). University is worth attending just to meet these people besides any other side benifit.

    I have had 3 jobs since finishing University but keep in touch with no-one I have worked with. Suffice to say work is not where you will probably make any friends at all.

    Here's the rub - this is how your life is going to be too. You are going to have very few friends and most people you talk too will be jerks. Almost all of your working life you are going to think what you are doing is a steaming pile of crap. If you go down the Degree, IT job, Team Leader/Sys-admin path you will get to about 25 and realise your life really, really, really means bugger all to anyone. 25 is the water shed that you realise if you don't move on and do something you feel passionate about you will be stuck doing crap for the rest of your life.

    Now, here's the blue pill...

    What do you want people to remember about you when you die. Seriously, what is the way you want to be remembered?

    Answer that question (for example, say you want to invent a flying car) and then take a look at where you are now (for example, just about to start University).

    Connect the dots and that's your "plan". Stick to it but be flexable when the situation demands it.

    Why do all this? What's the point? Because what we have left behind us when we are gone is all we ever were. So make the world a better place and do something you believe in. Because when it's 5.30 am on a Monday morning and your alarm goes off you need to be happy to get out of bed and go to work. If not, you will just end up as jaded and stupid as every one else.

    Good luck.

    T

    P.S. Seriously, go invent my flying car!!
  • Fuck a lot (Score:2, Insightful)

    by Anonymous Coward on Sunday June 13, 2004 @05:13AM (#9412026)
    I would like to add: fuck a lot, while you're still young and in some kind of shape. It doesn't hurt to stay in shape, either.

    There's nothing but head injury, cancer, stimulant use or the psychosis of religion that can keep you getting more functionally intelligent by the hour, but time'll also get you out of shape in a snap if you don't pay attention.

    It's a pain to get back up, after that has happened.
  • by DavidD_CA ( 750156 ) on Sunday June 13, 2004 @06:12AM (#9412170) Homepage
    There seems to be a lot of people putting their commentary down, but few people actually setting that aside to answer your questions.

    I'll tell you a few things that I've found helpful to myself, and why, and let you choose what works best for you. After all, everyone is different.

    Someone else here mentioned the book "How to win friends and influence people". I'd like to suggest going one further and actually taking the 12-week Dale Carnegie class that uses that book. It's once a week, 3 hours, 12 weeks total, and will bring you out of your introvert shell like nothing else will. I needed it, I went, and boy did it really change my life. Not to mention you'll make 10-15 friends from the class that you'll really bond with.

    Second, I suggest you look into your local city's Leadership program. Typically, the Leadership program is a one-year course where you sign up and become part of a group that meets once monthly. You are given all sorts of really facinating information about where you live, taken on tours, meet people, etc. You'll learn about the economics of the city, politics, agriculture, police/fire, education, industry, whatever. You will also typically put together some kind of philanthropic project to help your city. And, you'll meet tons of really important people who can help you out.

    Third, I recommend you get the stupid piece of paper called a degree. I put this off for 8 years and finally decided to do it. I'm taking online classes at University of Phoenix which allow me to go at my own pace, at home or wherever I have an Internet connection. It's far better for me than a stuffy school. Why get a degree? Because most people won't hire you for any decent job today without one. The online classes only suck up about an hour a day of my time -- far less than a real college, same benefit.

    I think it's great that you have so many interests. I recommend persuing them all to some degree. You love to learn and should make the most of it.

    If you're financially secure, don't try to get a full-time job just yet. Build yourself a checklist of all the things you want to accomplish in education and take care of that foundation first. Then persue your career -- after you know what you want to do.

    You should also consider taking at least a class each on business management and marketing. Most techie guys have no clue when it comes to either of these -- and if you are to be successful (for yourself or to a future employer) -- the information you'll get from even a beginning course will be invaluable to you.

    And, once you start earning an income, buy a house as fast as you can. This is about the most important financial advice anyone can give. I'll stop there 'cause it didn't sound like you were asking for this, but it's so important it had to be said.

    Someone else on here said that you can't change the world. Ignore them. They're just bitter that they tried and failed. Our history books and newspapers are full of people who have changed the world. I'm sure they were told the same thing.
  • Re:Advice (Score:3, Insightful)

    by sylvester ( 98418 ) on Sunday June 13, 2004 @06:59AM (#9412258) Homepage
    Do anything you want, enjoy it, and live off it.


    Work very hard suffering mentally at first, and eventually have big payoffs (psychologically and likely financially).

    Work very hard suffering mentally the whole way. Rewards come but you push them away because they just interfere with what's important.

    Resign yourself to become just a mindless peon.

    Your post hit a nerve with a few people, myself included. But it's not clear what choice you're suggesting is right -- or maybe none, but only the last one is wrong?

    It's hard to have a serious discussion about this on /., but essentially I've been pondering for a year or two how to balance my ambition (which sometimes seems to overwhelm me) with my ego (which I've been keeping in check since grade school when I realized I wasn't "Uniquely bright", just smart in a narrow way that society likes -- sorta).

    To believe that I should pursue my ambition, I (might?) have to give up some aspects of normal life. And I need to buy into my own ego, to believe that I *can* do the kind of things that I want to do. (To take it to the extreme, that would probably be going into politics, with the goal of going for something pretty high-up. Or starting a business, with the goal of changing how people think/buy/don't buy.)

    Cheers,
    Rob
  • Save a buck or two (Score:2, Insightful)

    by crashfrog ( 126007 ) on Sunday June 13, 2004 @07:03AM (#9412265) Homepage
    Don't go to an expensive college. If you're anything like me, and you sound like you are, you probably managed to slack your way through high school, learning much, much more outside the classroom in the process.

    Unfortunately you probably never developed much in the way of academic discipline, so you're going to succeed in every college arena except for grades.

    There's a very strong possibility you won't graduate this first time around - I didn't. But I went to a private school and racked up a shitload in loans.

    Don't be like me. This is going to be a great experience for you, and you'll love the college environment, but when you look back, everything you learned will probably have been outside the classroom. That's not a bad thing, but you're wasting tuition if you don't get a degree. And you probably won't, this time.

    So, if this is going to be throwaway college for you, don't blow the money on private school. Get into a cheap university, and by all means, live in the dorms. The best part of college for you is going to be outside the classroom; as a result, little you do in the classroom is going to matter this time. Save your money for the school you get into after you drop out of this one.
  • by sylvester ( 98418 ) on Sunday June 13, 2004 @07:12AM (#9412287) Homepage
    So all you dumbasses out there, you know, those that could never fabricate a circuit board, never write a line of Perl, or tell the difference between Everclear and a nice Amarone: Consider yourselves lucky. You live nice, happy, simple lives. Fulfilling lives. You feel good when you die.


    I see all the issues that you see. I feel bad when I don't buy my products carefully. I have what I have come to term "Affluent White Male Guilt." That makes me sound like a whinging middle-ager with a potbelly. I'm not.

    Lately, I've been thinking about ambition and ego. And I realized that some of this guilt that I have, and I'd guess you share, about not doing enough is because we can do so much. I'm comfortable in social situations, get along well with people, intelligent with computers. Those are my traits. But the rest is luck: I'm in the luckiest 100th of a percent in the world in terms of where I was born, when I was born, how I was raised. Wealthy enough to go to school, not so wealthy to be shielded by it. I've traveled, etc.

    All of this sounds like a giant ego trip, and to some extent it is, but the point is this: I think that those of us who, like me, are just that lucky, share a certain guilt. This is the guilt of being lucky, and knowing it, and knowing that you don't do enough with it.

    And I believe that is the curse that you speak of. It's not intelligence, but intelligence is part of the luck.

    -Rob, taking it one step further.
  • Re:Advice (Score:3, Insightful)

    by zors ( 665805 ) on Sunday June 13, 2004 @07:29AM (#9412310)
    Give me a fucking break.

    Is anyone else getting tired of this bullshit woe-is-me-life-is-pointless-nothing-can-change-bec ause-thats-the-way-the-world-is?

    If your life is shitty, change it, do something about it. Shitty things have been happening to people for thousands of years, yet society moves on.

    (i'm not just talking to people posting right here right now, this is more of a rant on society)

    And yes, you're right, i am a middle class white kid from a good home who's only angst has been mostly self created. I say what i say because i have faith in people. I KNOW that people can change, no matter what their age or personality type. You just have to make the choice and stick to it, all the reasons people have to not change is just a story we make up. Examine almost any problem a person might have in their life and it usually comes down to the choice they have to make, and a story; maybe its what people might think or how hard something could be or actually is.

    I just think we ought to give people more credit for their lives than we do, that its basically the luck of the draw with your social standing or personality type or their intelligence.
  • by smoon ( 16873 ) on Sunday June 13, 2004 @07:49AM (#9412336) Homepage
    Do something else. Be a missionary. Join the peace corps (do they take people w/out college degrees?). Hike the appalachian trail.

    Main point is to get your know-it-all attitude fixed. Can't focus on something for long periods of time? You _need_ to learn to stick with things or you'll never get anywhere in life.

    The real winners in our society are not the smartest, brightest people. They are the ones who through sheer effort of will and unwillingness to give up force success on themselves. This goes for anything - managers (even PHBs), athletes, government workers (ever seen the battery of tests and bureaucratic stuff these people live with?), startup businesses, rich people, poor people (yes, you can succeed at being poor if you try hard enough). Smartness helps -- becoming an M.D. takes more than hard work (lots of that though) -- but it's hardly the major determining factor for most things.

    So once you've figured that out, _then_ go to college. It doesn't much matter what the degree is in or where you get it from. People with college degrees are the aristocrats of the current era. Without a degree you can't go very far. With a degree (even a fine arts from a local college) you can get a lucrative position in a stable organization (assuming thats what you will eventually want, which it will be once you settle down and have kids).

  • by Anonymous Coward on Sunday June 13, 2004 @09:44AM (#9412673)
    I may be being thick but shouldn't that be:

    Things that are not black are not all ravens.

    Which would be true.
  • Re:Let's see (Score:3, Insightful)

    by AdamHaun ( 43173 ) on Sunday June 13, 2004 @09:56AM (#9412729) Journal
    No, it doesn't. What it means is that, if you want to deal with other people, you're going to have to meet them halfway. It's both unfair and unreasonable to expect other people to deal with a prima donna "genius". The end result of this adaptation is not Wal-Mart, it's getting past the work you don't like in order to get to the stuff you do.

    Put it this way. I'm an engineer. My boss walks in and hands me a project that I don't really want to do. Do I:

    A. Turn up my nose, refuse to do the work, and demand something different.
    B. Take the work, but remind my boss that I'm too good for this sort of thing.
    C. Try to get my coworkers to do the work. After all, they're just a bunch of drones, right? They're perfect for it!
    D. Take the work and do well on it, knowing that when a really cool project comes around I'll be more likely to get it.

    You can't live life without compromises. What you want is to pick the compromises that benefit you the most. If you want to deal with the world, sometimes you have to play by its rules.
  • Re:Advice (Score:5, Insightful)

    by troc ( 3606 ) <troc@ma[ ]om ['c.c' in gap]> on Sunday June 13, 2004 @10:02AM (#9412752) Homepage Journal
    Erm, 'ello. I'm a bit fick aledgedlee from wot you is sayin' abowt cognitiv'bilitee. Maybe I'l full of myself but it taysts liek a cheze towsti. Yum. Innit.

    Where was I. Slashdot has far more "up themselves" teenaged geeky nerds than you will get anywhere outside a Maths and Star Trek convention. Those people will, as is natural at 16 (and on to 15 or so) believe they are the dogs bollocks and that they are better than everyone else.

    However if you read the more mature, legible, sensible, well-thought-out, adult posts - which are probably from those of us who have "been there, done that" as it were, you'll realise that most of us (and I mean the human race in general) feel alienated, differently clever (ooh, so politically correct) or whatever at some point in our lives.

    For example at school I felt "superior" to most people around me. As an undergrad I realised what a twat I have been at school and that some people might actually be almost as intelligent as me (obvioujsly not quite as intelligent as their exam results were better :). When researching my masters I could look back and see what an immature person I had been as an undergrad and how my attitude towards people I felt were slower than me, or generally less intelligent (i.e. everyone) was a load of bullshit. I still harboured a secret knowledge that I was better than all the other people on the course.

    Then it came to my PhD - I was 25, I had travelled the world, lived away from home since I was 18 and I was much more secure and mature. I could look back on even my masters and see how conceited I was at 22! I was surrounded by people of the same intelligence as me - but in differen ways - which is why we were studying different topics! I felt properly grown-up at last and was over all the stupid angst and self-righteous feelings as before.

    Now I'm over 30 and I can look back and see that even at 25 I had a lot of growing left to do. I am sure that at 40 I will feel the same way .....

    To summarise. get over it :) Grow up. It might take 20 years but the journey is worth it.

    Troc.
  • Re:Advice (Score:5, Insightful)

    by RESPAWN ( 153636 ) <respawn_76&hotmail,com> on Sunday June 13, 2004 @10:28AM (#9412895) Journal
    have more or less relegated college to a chore at this point. I have a little less than 2 years left and I am taking some time off from it and working in my field. I am so much more successful in work environments that it is my sincere hope I won't ever have to go back. Basically college sucks IMHO. Aside from a handful of schools with extremely competitive admissions you are going to be surrounded by complete idiots. After a year you will stop wondering why everyone gets drunk, high, stoned, coked out of their mind every weekend because you will probably be doing it yourself.

    I can't say that I really enjoyed college all that much, but I do have to disagree about the comment regarding idiots. Maybe I was lucky and I just knew a lot of smart people, but the one thing that surprised me the most when I went to college was the sheer number of people there just as smart as I was. I was a lot like the poster in high school (in fact, I test as an INTP), probably smarter than most of the people I graduated with yet I cared so little that my GPA was only 3.6 or 3.7. (Despite having the singular distinction of taking more AP classes than anybody else I graduated with.) Frankly, it was a kind of a slap in the face when I got to college and realized that I might actually have to do some work if I wanted to compete academically.

    I have to disgree, however, that attending college is an entirely useless enterprise. Many employers won't even look at you if you don't have a college degree. Maybe it's true that, once you're in the field for a few years and have gained some experience, then you will find it easier to get jobs, but then you also run into some problems where some companies will refuse to promote people past a certain point if they don't have a college education. It may not be fair, but it happens.

    I'm not entirely certain that my college lived up to my expectations of what a place of higher learning should be, but I am still glad that I went. Don't look at college from the perspective that it will be giving you everything you need to be successful for the rest of your life. It won't. Just look at it as a means to help you get farther in life. Put in your time, and get your degree. Employers like to see qualifications such as college degrees. It makes them feel more confident that you will be able to do the job.

    Lastly, and I hate to sound so trite here, but you will grow more as a person while at college. Especially if you go away from home so that you are somewhat on your own and don't have your parents to use as a safety net. While at college you will grow and mature and learn a lot more about working with the different kinds of people that you meet out in the "real world."
  • A Reminder (Score:1, Insightful)

    by Anonymous Coward on Sunday June 13, 2004 @10:32AM (#9412914)
    Advice is just that, kid. Don't come back here in a year blaming Anonymous Coward or your Daddy or anyone for telling you what to do. You're your own person, no matter what anyone ever tells you. The onus is on you.

    Most of the assmasters here (IMHO) are jealous of you or have been fucked up by girls, school, drugs, whatever. Mostly they are talking to themselves, back in time, with whatever wisdom they think they've gained. Some of that wisdom could be wrong - we're talking about 25-49 career oriented bureaufucks. They're not trustworthy. They want you to join the tribe.

    The happy truth is, kid, there's more to life. And if you want to live a full life, here's a simple formula: do a little at a time and it will all add up. Patience actually works. You may find yourself wailing, you may find yourself poor and hungry and in the wilderness of our hideous society. But it's all experience, right?

    At some point you'll probably start questioning your beliefs, even isolating your limiting beliefs. You may have already done this; at any rate, if you've any brains you'll work out some things make you too happy, kind of like candy, and eventually make you sick. Some things are fulfilling, like vegetables. They're nutritious, good for the mind/body interface.

    My advice is don't take anything too seriously. I'll wager you've never been on the brink of death, never had the kind of life changing experience that makes that rule really meaningful. Eventually you will, and you'll find it handy.

    Don't take that rule too seriously, though. Rules are suggestions, consensus reality is just a consensus. If you really want to prove yourself, make your own life.

    Think of your experiences like a toolbox. You choose what you take from each experience, and learn modes of behavior that enhance or detract from your experience. With these you populate your toolbox. And for all intents and purposes, that's what you are.

    So be careful. The next ten years are usually pretty rough on a person.

    Best of luck, kid, and everyone else too.
  • Re:Advice (Score:5, Insightful)

    by Facekhan ( 445017 ) on Sunday June 13, 2004 @10:39AM (#9412941)
    Actually I tended to like my general education classes more than my major classes. At least in core classes I did not have to sit through things I already knew.

    A person may need a college degree out in the job world, but chances are they don't need a college "education". Most college graduates don't work in their major field and many don't even work in jobs that have any use for a degreed person at all.

    I attended a large private university and it probably does suck more than most schools as evidenced by its dismal 74% freshmen retention rate.

    People who don't like it when others bash college always have the same refrain "you are a know-it-all kid...." I love it when this comes from people who never even went to college themselves or from people who majored in bullshit. Yeah my grades are only ok but I certainly was not flunking out. One of the few classes I failed was the result of being the only one in the class who did not blatantly cheat. The professor left the room during exams and it was lets compare answer time.

    Why is it that everyone who feels they do not like college has to be wrong? Do you think colleges are run by ancient bearded men in robes who have already thought of everything? Colleges hire incompetent staff working for a slave wage except for the top administrators making hundreds of thousands dollars plus incredible personal benefits. Professors are almost all part-timers and TA's at most schools and the former get paid about the same rate as a highschool teacher in a poor district and the TA's barely get anything at all.

    Which brings me to my other piece of advice, DON'T pay for Grad school. Work at the school, do grad school for free, only suckers pay for it. If you only saw the kinds of easy jobs that grad students got at my school for which they worked 30 hours a week and got 9 credits of free grad school a semester. You know what a help desk support specialist does? She sits there and revises her resume for 6 months straight for which she is paid 5 thousand dollars worth of classes. The undergraduate technicians get 7 bucks an hour to drive their own cars around campus at their own expense and fix whiny girl's computers.
  • Re:Advice (Score:5, Insightful)

    by Facekhan ( 445017 ) on Sunday June 13, 2004 @10:46AM (#9412980)
    Most valuable college class: English Composition 1 & 2, closely followed by Speech.

    If you can write well, you can bullshit through almost anything. It really does help at work. Speech? Just make your first speech a practice run at asking for a raise and you see the point of not being nervous speaking in front of people. I am quite confident I will be getting a 50% raise tommorrow afternoon.

    Least Valuable college class: Philosophy of Science closely followed by Survey of computers in business (ie microsoft propoganda, bad tech info, and half a MOUS course. )

  • Re:No, I disagree (Score:3, Insightful)

    by deanj ( 519759 ) on Sunday June 13, 2004 @10:53AM (#9413013)
    In the time it would take the person to go from a BS degree to a PhD, they could have worked their way through to company and even gone past that level 17.

    Not to mention the years of income, experience for new jobs, etc.
  • by ccmay ( 116316 ) on Sunday June 13, 2004 @05:09PM (#9415266)
    Take Dufus in the Whitehouse for example. Few people would claim that he is an original thinker or highly knowledgable.

    Well, seeing as how he was able to fly a Delta Dart without killing himself, and subsequently earned an MBA from Harvard, I'd say he's got to be in the 95th percentile at least. His name may have got him into Harvard, but it sure didn't get him out.

    You would think that the Left would have learned long ago that the talent for glib public speaking is not at all equivalent to intelligence. After all, aren't they continually running down things like the SAT test, with claims that the ability to play basketball or tell stories or make friends is just another kind of intelligence like the ability to do calculus?

    On the other hand there are people with blistering high IQs and degrees in Nuclear Physics who can't find a job better than part time computer class instructor.

    Carter was a nuclear engineer and a dismal failure. He has made a pretty good ex-President, though.

    Clinton was a Rhodes Scholar and masterful public speaker whose major initiatives either failed spectacularly or were poached directly from the Republicans. He squandered his opportunity to make a lasting impact, and will be remembered by history as an easily-distracted lightweight who had the good fortune to be elected at an exceptionally peaceful and prosperous time.

    -ccm

  • Better yet... (Score:1, Insightful)

    by Anonymous Coward on Sunday June 13, 2004 @05:56PM (#9415508)
    Avoid school altogether. Go out into the world, try to make it without that piece of paper the world seems to hinge upon, get abused in more than one way, and then you'll have the motivation you need to attend school.

    Better still, if you make it, you will have circumvented the new opiate of the masses: education. At university prices, you'd probably be better off taking the five stacks of high society you'd spend there and start your own business. If you lose it, you're a young pup with plenty of time to start over.
  • by HeyLaughingBoy ( 182206 ) on Sunday June 13, 2004 @06:02PM (#9415537)
    Companies want employees that do everything they are told

    No, they don't!
    Companies -- and by that I mean the real decision makers, not just your boss -- want employees who make them richer. Pretty much all employees do what they're told; the ones that don't are an insignificant number and generally end up fired or marginalized. You can't distinguish yourself by showing up and just doing what you're told.

    You're right: he needs to "get over being clever," but also to decide if he wants to excel at something (in this case a job/career) and figure out how to apply his intelligence properly to that task.

    Hint: it usually involves going way beyond doing just what you're told. It requires that you figure out the next step and do it, or suggesting an improvement that fits the company's business model, not just something that makes you look like you're trying to be smart.

    Problem is that most geeky types -- yours truly included :-( -- take a long time to learn that no one cares if they're smart. People care what you can do for them and that you can do it well without making them feel stupid. That's a hell of a lot more important to your success than just being clever.
  • Re:Advice (Score:2, Insightful)

    by bzipitidoo ( 647217 ) <bzipitidoo@yahoo.com> on Monday June 14, 2004 @01:41AM (#9417566) Journal
    disciplined, tiring, rigorous work

    Why is it "work" has to be unpleasant? Granted, can't see what could be pleasant about toiling 80 plus hours a week in a sweatshop that makes clothes. Part of the definition,I guess-- play is the antonym of work. Need some new definitions; it's poisonous to think work and fun are exclusive. Seems too many managers think they can make the help deliver more and faster by pushing and stressing them to the ragged edge. If you're having fun, then you must not be working hard enough, never mind how productive you are. This exclusive thinking manifests as feeling that time "wasted" on fun could've been used to do work. All that's not to deny that getting things accomplished often does involve tedious, boring, repetitive or otherwise unpleasant activities, but those activities ought to be secondary to work one enjoys.

    Skip the Alaskan fishing boat "no pain no gain" nonsense. Doing the #1 most dangerous job in the world is "no pain no gain" on steroids. I hear all the time about people (type Ts?) who find their life meanings that way. Are you that sort? No? It works for them; great. Doesn't mean it works for ordinary mortals.

    If you really are so uniquely bright, try to parlay some of that into wisdom. Life doesn't have to suck, and work doesn't have to be solely for the paycheck.

  • by stgabriel ( 787316 ) on Monday June 14, 2004 @06:46PM (#9424540) Homepage
    Actually, intelligence isn't necessarily genetic, its a matter of training and mental exercise. Perhaps the most important thing to realise, if you're intelligence, is ones own shortcomings. Intelligence can be powerful for learning, but doesn't imply 'good judgement'. I think the most important advice is to cultivate 'character' and realise that in seeking out people to 'be around', that good character and judgement (or wisdom) is much more important than simply intelligence. Intelligence and curiosity has value, but knowing how to live and be accepting and tolerant are more important skills. Learning to be happy is an untaught skill, learning how to find ones ego as 'boring and misleading' is another- but far more important than most others. Also, diversify your knowledge base, and read avidly, look into the past and read good literature through your life. You'll find it reassuring and redemptive.

Those who can, do; those who can't, write. Those who can't write work for the Bell Labs Record.

Working...